Gastroenterology #2

Lakukan tugas rumah & ujian kamu dengan baik sekarang menggunakan Quizwiz!

You are asked to follow up with a 47-year-old female who was recently hospitalized for pancreatitis associated with a biliary stone. The stone passed spontaneously, and the patient was treated with analgesics and IV fluids until she was able to resume oral intake. She was doing well until earlier this morning when she experienced increasing abdominal pain and feverishness. The patient continues with a temperature of 102.8° F (39.3° C). CBC revealed WBC of 18.9 x 10 3 /mm 3 . An urgent CT scan of the abdomen was ordered and shows evidence of early pancreatic necrosis. The most important intervention at this time would be: A) Blood cultures and broad-spectrum antibiotics B) Endoscopic retrograde cholangiopancreatography (ERCP) C) CT-guided aspiration of the pancreatic necrotic tissue D) Emergent surgical consult

Answer D. Emergent surgical consult Explanation Infection of necrotic pancreatic tissue is a serious, life-threatening complication of pancreatitis. Insults to the pancreas leading to acute pancreatitis include obstructive pathology of the common bile duct, alcoholism, and trauma. ERCP can be helpful in the diagnostic phase and may be useful to remove gallstones, but otherwise has a very limited role in the diagnosis or treatment of pancreatic necrosis. Blood cultures and antibiotics are important tools in the management of pancreatic infections but neglect the debridement and drainage of necrotic pancreatic tissue. CT-guided aspiration is an effective tool for pancreatic abscesses and pseudocysts but is inadequate for diffuse pancreatic necrosis. Surgical intervention can be localized versus extensive depending on the amount of tissue involved.

A 54-year-old male presents as a new patient for refills of his omeprazole prescription. He has been taking this medication for 3 years with very good control of his symptoms. He reports that an upper GI series 3 years ago demonstrated a sliding hiatal hernia. He is otherwise well with no social risk factors. Of the following, the most serious consequence of a persistent hiatal hernia is: A) Aspiration pneumonia B) Stomach cancer C) Pernicious anemia D) Esophageal cancer

Answer D. Esophageal cancer Explanation A hiatal hernia decreases the effectiveness of the cardiac sphincter in preventing gastro-esophageal reflux. The increased reflux raises the incidence of Barrett esophagus, which leads to an elevated risk for esophageal cancer. Reflux is associated with reactive airway disease, but in a neurologically intact patient with no history of drug or alcohol abuse, aspiration pneumonia risk is not significantly elevated. Atrophy of gastric mucosa leads to pernicious anemia, but hiatal hernia does not directly alter the risk for B 12 deficiency. Risk for stomach cancer is not significantly affected by the presence of a hiatal hernia.

A 76-year-old man presents with new-onset dysphagia. He describes his dysphagia for solids and liquids for about the past 4 weeks. He has no prior history of dysphagia. He also describes nocturnal regurgitation of sour food. Significant for COPD from long-standing tobacco use (60 pack years). He has stable dyspnea on exertion and a chronic cough, but has recently developed a nocturnal cough. He has lost 20 pounds over the last month. He denies chest pain. RR 18 bpm Mild cachexia Barrel-shaped chest with an increased expiratory phase; scattered bilateral wheezes. The abdomen is normal. Barium swallow: Dilated esophagus with bird beak narrowing of the distal esophagus. Which of the following is the most appropriate next step in diagnosis? A) Start empiric omeprazole and reevaluate in 4 weeks. B) Schedule Botox® injection. C) Esophageal motility. D) Esophagogastroduodenoscopy (EGD). E) Refer to surgery for laparoscopic myotomy.

Answer D. Esophagogastroduodenoscopy (EGD). Explanation With the new onset of symptoms, advanced age, and weight loss, one is suspicious that this is pseudoachalasia (or "secondary achalasia")—clinically the same as achalasia but caused by a malignancy. Usually the malignancy is at the gastroesophageal junction. The x-ray and manometric findings may be the same as classical achalasia. However, in this case, this patient definitely needs an EGD to look for any suspicious mass. Esophageal motility is helpful in confirming the diagnosis of achalasia, but of less importance in this specific patient. One should definitely confirm the diagnosis before proceeding with treatment options like Botox® or surgery.

A 45-year-old man presents with recently diagnosed peptic ulcer disease. His ulcer was found on EGD and was in the 2nd portion of the duodenum. Also on EGD, he was noted to have fairly significant esophagitis. He had presented with an upper gastrointestinal bleed yesterday. On further questioning, he relates that he has been doing well except for dyspepsia on occasion after eating "spicy" foods and has noted increasing episodes of diarrhea for the last few months. PAST MEDICAL HISTORY: Noncontributory FAMILY HISTORY: Mother with peptic ulcer disease in her mid-40s, died at age 50 of severe upper GI bleed Brother Hx of peptic ulcer disease SOCIAL HISTORY: Works as a carpenter Smokes 1 ppd x 30 years Alcohol: 6 pack of beer on weekends REVIEW OF SYSTEMS: Negative for fever, chills Denies night sweats No tachycardia No vomiting No burning on urination No hair loss or changes in hair character PHYSICAL EXAM: Vital signs: BP 140/50, P 88, RR 15, Temp 98.7° F, Ht 5' 5", Wt 170 lbs HEENT: PERRLA, EOMI TMs clear Throat clear Neck: Supple, no thyromegaly Heart: RRR with murmurs, rubs, or gallops Lungs: CTA Abdomen: Bowel sounds present in all 4 quadrants, slightly hyperactive; no hepatomegaly, spleen tip non-palpable, mid-epigastric tenderness to deep palpation; no rebound Extremities: No cyanosis, clubbing, or edema Skin: No rash Rectal: Heme-positive GU: No lesions noted; normal male genitalia LABORATORY: A rapid urease test was done on the biopsy specimen and was positive for H. pylori. Based on his history and findings at EGD, which of the following tests or diagnostic studies should you order now? A) Repeat EGD in 5 days. B) Computed tomography of the abdomen to rule out malignancy. C) Follow-up H. pylori serology in 1 month to look for cure. D) Fasting serum gastrin level. E) Barium swallow.

Answer D. Fasting serum gastrin level. Explanation Several issues should make you think about Zollinger-Ellison syndrome (ZES). Note that his ulcer is in an unusual location; generally, any ulcer past the duodenal bulb should make you think about ZES. Also, he has severe esophagitis, which can be seen in regular ulcers; but the finding of the EGD makes ZES more likely. Additionally, his family history is quite strong, and he has been having diarrhea—another hallmark of this syndrome. Although diarrhea often occurs concomitantly with acid peptic disease, it may also occur independent of an ulcer. Etiology of the diarrhea is multifactorial, resulting from marked volume overload to the small bowel, pancreatic enzyme inactivation by acid, and damage to the intestinal epithelial surface by acid. Occasionally, you can have mild malabsorption of nutrients and vitamins. The diarrhea may also have a secretory component due to the direct stimulatory effect of gastrin on enterocytes or the co-secretion of additional hormones from the tumor, such as vasoactive intestinal peptide. Gastric acid hypersecretion is responsible for the signs and symptoms observed in patients with ZES. Peptic ulcer is the most common clinical manifestation, occurring in over 90% of gastrinoma patients. Other clinical situations that should create suspicion of gastrinoma are ulcers refractory to standard medical therapy, ulcer recurrence after acid-reducing surgery, or ulcers presenting with frank complications (bleeding, obstruction, and perforation).

A 24-year-old male presents to the Emergency Department with a 2-day history of severe nausea, vomiting, diarrhea, and fever (temperature up to 101° F). Before this illness, he was healthy and active without any underlying medical problems. His girlfriend, who is a nurse, brought him to the ED when she suspected that he was dehydrated. He does not take any medications on a regular basis, although for the past few days he has been taking up to 8 extra strength acetaminophen (500 mg) per day. Family History: Unremarkable Social History: Significant for unprotected sex with several females over the past 4 years. He does describe ingestion of raw oysters one week prior to admission. He consumes alcohol—3 beers per week. Physical Exam: The patient is in moderate stress. BP 100/70, HR 100. His skin turgor is decreased. There is obvious scleral icterus on exam. Cardiac and lung exam is otherwise normal. The abdomen is soft and non-distended with normal bowel sounds. Extremities are normal. Laboratory: Significant for WBC 5,500 and Hgb of 15.5. The differential is normal. Total bilirubin is 4. AST 30, ALT 28, alkaline phosphatase 90, INR 1. Which of the following is the most likely explanation for the patient's jaundice? A) Acetaminophen toxicity B) Hepatitis A C) Autoimmune chronic active hepatitis D) Gilbert syndrome E) Hepatitis B

Answer D. Gilbert syndrome Explanation This patient likely has a viral gastroenteritis. The stress of this illness combined with dehydration has led to the uncovering of the Gilbert syndrome. If one were to analyze the bilirubin, one would see that it is all unconjugated (indirect). Despite his intake of raw oysters, which is recognized as a risk factor for hepatitis A, if jaundice was from hepatitis A there would be abnormal transaminases. There is a syndrome of prolonged cholestasis after an acute hepatitis A episode, but there would be a longer prodrome of symptoms than in this case. And even in this case there would be elevation of the alkaline phosphatase. Likewise, hepatitis B would have elevated transaminases. Acetaminophen toxicity would have a very high elevation of transaminases. It is possible to have acetaminophen toxicity with doses as low as 4 gm a day; however, this occurs only in patients with severe malnutrition and/or alcoholism. Finally, autoimmune chronic active hepatitis, which is unusual in men, would also have elevated transaminases. The key to this case was the normal transaminases.

A 19-year-old lifeguard at the local water park in your area presents for his final routine hepatitis B vaccine before going to college. Recently, there has been an outbreak of diarrhea at the water park confirmed as E. coli O157:H7. He is concerned about the diarrhea at the park and asks what he can do to limit his exposure. You explain that the outbreak has been linked to hamburgers at the park that were undercooked. He is concerned because he eats hamburgers twice daily. You explain that it is unlikely that he will become ill but to call you at the first sign of diarrhea. The next morning, you receive a call from him saying that he has diarrhea. It is bloody in character. You tell him to come in right away. He arrives and is ill appearing. He says the diarrhea started early this morning, and he has gone about 5 times since. He noted large amounts of blood in the initial stool, but it has since tapered off to a few streaks. He says he is lightheaded and dizzy. PAST MEDICAL HISTORY: Healthy SOCIAL HISTORY: As above Doesn't smoke Doesn't drink alcohol Not sexually active FAMILY HISTORY: Non-contributory ROS: Fever, chills noted early this morning Weight loss of approximately 5 pounds since yesterday, he thinks Dizzy when standing No vomiting PHYSICAL EXAMINATION: Lying down: BP 120/80, Pulse 80 Sitting up: BP 100/65, Pulse 110 Temperature: 99.5° F, RR 18 HEENT: PERRLA, EOMI Tympanic membranes clear Throat clear; mucous membranes dry Neck: Supple, no bruits Heart: RRR without murmurs, rubs, or gallops Lungs: CTA Abdomen: Benign, no tenderness, no hepatosplenomegaly Extremities: No cyanosis, clubbing, or edema GU: Deferred Neurological: Reflexes equal and symmetrical throughout; no sensory or motor deficits noted LABORATORY: All pending Assuming that this is going to be invasive diarrhea with E. coli O157:H7 and after starting supportive fluid therapy, which of the following is the best treatment for his infection? A) Ceftriaxone 1 gm IV q day. B) Await sensitivities before starting therapy. C) Penicillin 3 million units IV q 4 hours. D) Give supportive care only. E) Ciprofloxacin 400 mg IV bid.

Answer D. Give supportive care only. Explanation Starting antibiotics in patients with E. coli O157:H7 actually increases the risk of HUS (hemolytic uremic syndrome). Therefore, supportive care is all that is indicated for this patient. Antibiotics are absolutely contraindicated!

A 68-year-old male has been in the ICU for 2 weeks with a very complicated story. He was originally admitted to the hospital for elective resection of a colon cancer of the cecum. Within the first three days after surgery, this was complicated by an aspiration pneumonia and acute respiratory failure, requiring intubation and mechanical ventilation. He also developed acute renal failure and a prolonged ileus, requiring the patient to be on TPN for nutrition. During this time, the patient has been kept on broad-spectrum antibiotics, including ticarcillin/clavulanic acid and gentamicin. The abdomen has been distended the entire period. The patient is unable to complain since the day they started ventilatory support. Now after one week of being afebrile, the patient has a fever to 102° F. Abdominal ultrasound shows a gallbladder with a thickened wall and a small amount of fluid around the gallbladder. No stones are seen. Laboratory: Bilirubin is 1.6, AST 110, and ALT 90. Given this scenario, which of the following is the most appropriate next step? A) ERCP and stent placement. B) Laparoscopic cholecystectomy. C) Change antibiotics to cover potential fungal sepsis. D) Interventional radiology to place a percutaneous cholecystostomy drainage tube

Answer D. Interventional radiology to place a percutaneous cholecystostomy drainage tube. Explanation This is a typical case of acalculous cholecystitis. It occurs in ill, hospitalized patients who often have multi-organ failure and are on TPN. The diagnosis is often delayed because of the atypical presentation and because the patient may not be able to complain (as in this case). The diagnosis is generally made by ultrasound showing a thickened wall of the gallbladder and fluid around the gallbladder. Though surgery can be appropriate, it often requires a surgically open technique rather than a laparoscopic technique because of the friability of the thickened wall. In a very ill patient, like above, it is appropriate for the radiologist to insert a catheter to drain the fluid out of the gallbladder. This can give initial improvement and allow surgery to be done in a more opportune time.

A 40-year-old man undergoes endoscopy for heartburn and dysphagia. At endoscopy, he is found to have 3 small duodenal ulcers and LA class D reflux esophagitis. He is placed on esomeprazole. Three weeks later, he undergoes fasting serum gastrin testing. His gastrin level is 600 pg/dL (upper limit normal is 110 pg/dL). He returns to your clinic. What do you tell him about his gastrin elevation? A) It is likely that he has atrophic gastritis. B) It is likely that he has gastrinoma. C) It is likely from primary G-cell hyperplasia. D) It is likely from taking esomeprazole.

Answer D. It is likely from taking esomeprazole. Explanation This intermediate gastrin measurement is likely from PPI use, which elevates the pH and causes gastrin levels to increase. If suspected, most test initially for gastrinoma before starting the PPI, which alleviates this issue. If the patient is on a PPI, you can stop it for a week, but in a patient like this where there is some suspicion of gastrinoma, it may be prudent to keep him on the PPI and proceed to the next test, a secretin stimulation test—which looks at rise in gastrin levels after secretin stimulation. It is thought that the test can be interpreted while on PPIs, although this is somewhat controversial. An initial fasting gastrin value above 1,000 pg/dL in a patient with stomach pH < 5 is virtually diagnostic.

A 25-year-old woman has a 3-year history of diarrhea and abdominal cramping. These symptoms seem to worsen with stress, but the diarrhea can wake her. She was told she had irritable bowel syndrome and was placed on a high-fiber diet. She was also given dicyclomine as an antispasmodic agent. However, her symptoms did not improve with these methods, and she is seeking another opinion. Past Medical History: Negative, and her only medicine is dicyclomine. Family History: Negative Social History: Unremarkable Review of Systems: Significant for a 4-lb. weight loss in the past 3 months. She denies any fever or chills. Physical Examination: She is in no apparent distress. HEENT: Unremarkable Chest: Clear Cardiac: Normal Abdomen: Nondistended with normal bowel sounds. She does have focal tenderness with some degree of fullness in the R lower quadrant. You order a colonoscopy, and the colon is normal until entering the right colon. At this point, one sees numerous shallow ulcers. The ileum is edematous with multiple shallow ulcers. Biopsies of these areas reveal chronic inflammation and ulceration. Some of the biopsies from normal tissue adjacent to the ulcers come back with normal histology without any significant inflammation. The most appropriate treatment at this time would be which of the following? A) Infliximab 300 mg infused over 2 hours B) Azathioprine 50 mg PO once daily C) Prednisone 40 mg qd with a taper of 10 mg every week D) Mesalamine 400 mg PO tid

Answer D. Mesalamine 400 mg PO tid Explanation This 25-year-old woman has a new diagnosis of Crohn disease. The involvement of the right colon and ileum is a very typical pattern of involvement. Although the pathologist does not describe granulomas, the chronic inflammation and ulceration in this setting would be considered diagnostic. It's not uncommon for these patients to have symptoms for several years before the diagnosis is made, as was the case in this patient. Regarding treatment, there is no reason why one cannot start with the most benign treatment, from a safety standpoint, and use mesalamine. She may respond well and become symptom-free. If there was more urgency, in that the symptoms were of greater severity, then one could use prednisone. Azathioprine can be very helpful in the patient who is requiring frequent courses of steroids, but her disease may not be severe enough to require immunosuppressive therapy. Likewise, infliximab can be helpful in the moderately severe cases of Crohn disease, but this may not be additionally needed.

A 32-year-old man presents for a physical to obtain life insurance. He reports being in good health generally. He takes no medications. He is a 1/2 ppd smoker and reports alcohol use of 1-2 drinks per day. On review of systems, he reports frequent heartburn for the last several months. He denies diarrhea. On physical exam, he is afebrile, blood pressure is 115/70, pulse rate is 72/min, respiratory rate is 16/min; BMI is 28. His abdominal exam reveals a non-distended abdomen without tenderness or hepatosplenomegaly. Laboratory studies: Na: 138 K: 4.0 Cr: 0.8 Glucose: 85 Hemoglobin: 11.0 g/dL Platelet count: 230,000/uL Alanine aminotransferase: 30 units/L Aspartate aminotransferase: 25 units/L Alkaline phosphatase: 80 units/L Bilirubin: 1.0 mg/dL What is the next best step in the management of this patient's heartburn? A) H. pylori serology. B) Trial of a proton pump inhibitor. C) Ambulatory pH study. D) Perform endoscopy. E) Tissue transglutaminase IgA Ab.

Answer D. Perform endoscopy. Explanation This patient presents with symptoms characteristic of GERD for the last 3 months. In performing an evaluation on patients with GERD symptoms, it is important to recognize "alarm symptoms," which include dysphagia, anemia, weight loss, and vomiting. This patient has anemia noted on his laboratory studies. The next best step would be upper endoscopy. Had alarm symptoms been absent, an empiric trial with a PPI would be appropriate. An ambulatory pH study would be indicated in patients with GERD who do not respond to empiric therapy. Testing for H. pylori would be indicated in patients with dyspepsia (burning, epigastric pain, bothersome postprandial fullness, early satiety). Tissue transglutaminase IgA Ab would be indicated to evaluate for celiac disease. Manifestations of celiac disease can include abdominal pain and anemia. However, celiac disease should present with signs of malabsorption, which this patient does not have.

20-year-old woman presents with a history of chronic diarrhea for over a year. She has complained of diarrhea on many visits to your office. Listed below is the laboratory work that has been done to date, including a few tests done today. The diarrhea is intermittent in character, lasts 3-5 days, and then her stools gradually return to normal. She has not noticed any blood in the stool. She has no nausea or vomiting. She has no other health problems. No one that she lives with (her boyfriend and 1-year-old) have problems with diarrhea. She has not had significant weight loss. The stools are not foul smelling and are usually fairly watery in character. PAST MEDICAL HISTORY: History of depression at age 16 requiring hospitalization; since then doing well on fluoxetine 20 mg q day SOCIAL HISTORY: Beauty college student Smokes 1/2 pack day for 3 years Alcohol—doesn't drink FAMILY HISTORY: Mother healthy, no health problems Father healthy, no health problems Sister anorexia Brother bulimia REVIEW OF SYSTEMS: No fever, chills No sore throats No increased nervousness No chest discomfort No wheezing No stomach pain No rashes No travel PHYSICAL EXAMINATION: Well-appearing WF BP 120/60, P 64, RR 18, Temp 98.3° F HEENT: PERRLA, EOMI, sclera non-icteric Neck: Supple; no masses Heart: RRR without murmurs, rubs, or gallops Lungs: Clear to auscultation Abdomen: Mild epigastric tenderness to deep palpation Extremities: No cyanosis, clubbing, or edema GU: Heme-negative stool; scant amount of stool in vault LABORATORY: CBC x 2 normal Electrolytes normal x 3 Liver transaminases normal x 2 Stool ova and parasites and fecal leukocytes x 3—all negative C. difficile toxin negative Chem 20—all within normal limits TSH normal T4 normal Gastrin level: normal Giardia-specific antigen negative Sigmoidoscopy negative Sodium hydroxide added to stool: turns red Which of the following do the history, physical, and laboratory findings suggest? A) Irritable bowel syndrome B) Need to proceed with colonoscopy C) Carcinoid D) Phenolphthalein abuse E) Bisacodyl abuse

Answer D. Phenolphthalein abuse Explanation Note that she has a normal physical examination, and we have spent quite a bit of time and money on diagnostic tests, which all have been negative. The only positive laboratory value is the sodium hydroxide test, which indicates that she is abusing phenolphthalein. You can also confirm this with specific urine tests for this agent. The bisacodyl could be confirmed also by urine testing. Irritable bowel syndrome is a diagnosis of exclusion, and we have found another etiology for her symptoms. Carcinoid is a rare cause of diarrhea, but she does not have any of the other symptoms such as flushing, tachycardia, and explosive diarrhea. Colonoscopy and EGD are reserved for the later stage of workup of chronic diarrhea and are not indicated at this point, particularly in light of our findings of phenolphthalein abuse.

A 55-year-old woman presents as a new patient for a routine checkup. On questioning, you determine that she has been relatively healthy and has not required any hospitalizations since the delivery of her last child over 20 years ago. She notes that she recently has had swelling of her hands and occasionally of her feet. She relates this to "eating too much salt." Her only other complaint is that she has frequent heartburn, and occasionally notes "food sticks when it goes down." She does not report any other problems. PAST MEDICAL HISTORY: As above, noncontributory SOCIAL HISTORY: Works as a 4th grade elementary teacher Quit smoking 30 years ago Alcohol: Drinks on occasion with her church group Married; husband is a forester FAMILY HISTORY: Mother still living; has hypertension, had CVA 3 years ago Father died 2 years ago from emphysema Sister diagnosed with lupus 10 years ago REVIEW OF SYSTEMS: Skin: On questioning, she notes that when she is outside in the winter her hands turn white and occasionally even have a bluish color to them GI: Recurrent reflux on a regular basis "Food sticking" as described above Occasional diarrhea Eyes: She notes dryness of her eyes on occasion No visual changes have been noted General: Mild weight gain in the last year GU: Vaginal dryness noted in the last year Has noted dyspareunia in the last few months PHYSICAL EXAMINATION: Well-developed, well-nourished woman in no distress BP 120/70, P56, RR 18, Temp HEENT: Noted that she has difficulty opening her mouth fully—says she can't open very wide PERRLA, EOMI, Discs sharp Tympanic membranes intact Teeth normal in appearance Throat clear; tonsils present Neck: Supple, no bruits Heart: Slight bradycardia, RRR with no murmurs, rubs, or gallops Lungs: CTA Abdomen: Bowel sounds present in all 4 quadrants, no tenderness to palpation; no hepatosplenomegaly Extremities: Skin over fingers is shiny Skin creases and hair follicles are not present No cyanosis, clubbing noted No noticeable joint abnormalities GU: Dry vaginal mucosa noted; otherwise nothing abnormal LABORATORY: CBC: WBC 8,700 103/mL with normal differential Electrolytes: WNL ESR: 48 mm/hour ANA present in nucleolar pattern at 1:640 Which of the following can be predicted with regard to her esophageal disease? A) She will gradually improve over time without medical therapy. B) The gastrointestinal findings with her illness will be usually confined to the esophagus. C) Treatment with H2 blockers will provide complete recovery from the esophageal disease. D) Progression is likely to occur, and stricture formation with nearly complete loss of peristalsis will be seen in later forms of this disease. E) It is unlikely that her esophageal disease is related to her skin findings.

Answer D. Progression is likely to occur, and stricture formation with nearly complete loss of peristalsis will be seen in later forms of this disease. Explanation This woman has scleroderma. Over 80% of patients have involvement of the esophagus. It causes reduced-to-absent lower esophageal sphincter pressure (unlike achalasia). The other symptoms and signs that she has go along with the disease. The symptoms will not improve as the disease progresses. H2 blockers may provide some symptom relief but are unlikely to provide complete recovery from this progressive connective tissue disease. Her skin findings are related to her esophageal disease. The gastrointestinal findings frequently involve other areas besides the esophagus.

A 70-year-old Asian male who hasn't seen a doctor in 20 years comes to your office. No significant history; he is brought in by family who reports that the patient is not eating and is losing weight. Patient says he has progressive difficulty swallowing. Six months ago, he could swallow soft foods, but now it is slow even getting liquids down. There is a long history of cigarettes and alcohol. Exam shows muscle wasting. Labs are significant for Hgb of 10 and MCV 72. A barium swallow shows a 3 cm segment of severe narrowing of the distal esophagus. Which is the next best step in the management of this patient? A) Give samples of a PPI. B) Esophageal motility study. C) Calcium-channel antagonist. D) Refer to GI for EGD.

Answer D. Refer to GI for EGD. Explanation There is high likelihood that this is esophageal cancer, most likely squamous cell cancer (long history of cigarette use). The next step would be EGD with biopsies to confirm the diagnosis. He does not have diffuse esophageal spasms, so calcium-channel blockers are not indicated. PPIs are not indicated. An esophageal motility study is not going to give you the diagnosis—you need tissue!

Which of the following is true regarding the use of corticosteroids in the treatment of ulcerative colitis? A) Active steroid use decreases the risk of malignant degeneration. B) Corticosteroids are a principal agent to maintain remissions. C) Rectal steroids avoid adrenal suppressive effects. D) Risk of avascular necrosis is increased with steroid usage.

Answer D. Risk of avascular necrosis is increased with steroid usage. Explanation Rectally administered glucocorticosteroids have a significant absorption and can lead to adrenal suppression. Steroids can play a prominent role in controlling acute inflammatory flares but have no role in maintenance control. Steroid dosing should be tapered to discontinuation as soon as tolerated by the patient. Despite their beneficial effects, continued steroid use also entails many side effects. These include adrenal suppression, osteoporosis, avascular necrosis, and glucose intolerance. Steroids can help control symptoms and inflammation, but they do not prevent the near inevitable progression to neoplastic degeneration associated with ulcerative colitis.

A 70-year-old male patient has absolutely no complaints. He is in for his routine checkup. At this time, he is found to have a Hgb of 10.7 with an MCV of 72. He denies any weakness or fatigue. PAST MEDICAL HISTORY: Significant for hypertension; his only medication is lisinopril SOCIAL HISTORY: Noncontributory with no alcohol or tobacco history; he is active and walks every day FAMILY HISTORY: Negative for gastrointestinal disease REVIEW OF SYSTEMS: Denies any abdominal pain, reflux, or change in bowel movements; he has never had any overt bleeding PHYSICAL EXAMINATION: Healthy 70-year-old patient HEENT: Normal Chest: Clear and has a normal cardiac exam Abdomen: Soft and nontender without organomegaly; his stool is brown and heme-negative You schedule a colonoscopy, which reveals moderate diverticulosis in the sigmoid colon. There is a single 5-mm AVM found in the cecum. There is also a 5-mm sessile rectal polyp, which was removed. At this point, which of the following would you recommend? A) Place patient on estrogen therapy to stop the bleeding from the AV malformation. B) Administer an H2 blocker initially. C) Elective sigmoid resection, because diverticulosis was the most likely cause of bleeding. D) Start patient on iron and monitor the Hgb every 2 months. E) Endoscopic ablation of the AV malformation, because this was the likely cause of anemia.

Answer D. Start patient on iron and monitor the Hgb every 2 months. Explanation This gentleman likely was bleeding from the AVM found in the cecum. The diverticulosis that was found was not the cause of the bleeding, because this does not cause occult bleeding. Diverticular bleeding generally cause an acute bleed with significant hemorrhage, resulting in maroon stool. Likewise, the polyp is too small to cause any bleeding. However, even though the AVM was the likely cause of bleeding, at this point it has not resulted in significant blood loss such that the patient is symptomatic. It is likely that this patient can be placed on iron and his Hgb will stabilize. He may slowly ooze blood from time to time, but the iron will replace those losses. Estrogen therapy is not of proven value for AV malformations, and it's not tolerated by men because of the side effects. H2 blockers are not indicated in AVM treatment.

A 27-year-old woman presents with a 3-month history of rectal bleeding, abdominal cramps, and diarrhea. Colonoscopy was performed, leading to a diagnosis of ulcerative colitis. She is started on mesalamine at a dose of 2.4 g/day. There is some initial improvement in her symptoms, but 3 weeks later, she develops a different abdominal pain: a severe epigastric pain that radiates to the back. The pain is associated with nausea and vomiting. She came to the Emergency Department, where her amylase was found to be 3,000 with a lipase of 2,400. Acute pancreatitis was diagnosed. The patient was made NPO, an NG tube was placed, and IV fluids were administered. Mesalamine was stopped. The ultrasound done at that time showed a normal gallbladder. After 5 days in the hospital, the patient was discharged with resolution of her symptoms and a normal amylase and lipase. Two weeks later the patient again presents to the Emergency Department with epigastric pain. This is somewhat similar to her past pancreatitis but not as severe. Amylase is 300 and lipase is 280. CT scan shows a 4-cm cyst in the body of the pancreas. Which of the following do you recommend? A) ERCP now to rule out common bile duct stone. B) Surgery now to remove the cyst and the gallbladder. C) Interventional radiology to drain the pseudocyst. D) Treat with conservative measures like bowel rest and IV fluids.

Answer D. Treat with conservative measures like bowel rest and IV fluids. Explanation This is a typical presentation of a pseudocyst: a recurrence of pain and elevation in the pancreatic enzymes several weeks after an acute event. One should never do ERCP in this setting because there will be risk of inducing an infection of the fluid of the pseudocyst. It is also too early to do surgery or percutaneous drainage of this acute pseudocyst. The recommended treatment is supportive care with follow-up of the cyst by CT scan to hopeful resolution. What was the etiology of this patient's initial pancreatitis? It may well have been secondary to the mesalamine. Mesalamine is one of the many drugs that have been associated with pancreatitis. Mesalamine probably should not be restarted in this case. It's also possible that, as in many cases of pancreatitis, the etiology is a gallstone blocking the common duct. This can be present even despite a previously negative ultrasound.

A 60-year-old Asian male who immigrated to the United States in 1950 was recently diagnosed with gastric carcinoma that is distal in character. He has started adjuvant combination chemotherapy. His oldest son comes in with his father today and asks what things he, the son, can do to "prevent getting this type of cancer." All of the following are risk factors for gastric carcinoma except: A) Blood type A B) H. pylori infection independent of known ulcer disease C) Diet low in fruits and vegetables D) Pernicious anemia E) Alcohol consumption

Answer E. Alcohol consumption Explanation Note that alcohol consumption is not a risk factor for gastric carcinoma. All of the other factors listed are associated with increased risk. Also remember that history of gastric ulcer is not associated with increased risk of carcinoma. Other factors that are associated include diets high in dried, smoked, and salted foods, as well as foods rich in nitrates, Barrett esophagus, and distal gastrectomy. Uranium mining is yet another association ... so if they describe someone who is a uranium miner, look for the possibility of gastric carcinoma. Finally, lower socioeconomic class is associated with increased risk.

A 60-year-old patient has been suffering with diarrhea for the past 2 years. She has undergone extensive evaluation, which has been negative. She's had 2 colonoscopies, including one with random biopsies, both of which were negative and otherwise unrevealing. The diarrhea is described as up to 6 watery bowel movements per day. It has been somewhat progressive over the past 2 years. 3 days ago, the patient started having nausea and profuse vomiting to the point that she could not hold down any food. Past Medical History: Otherwise unremarkable. The patient is not on any medications. Review of Systems: Positive for a long history of heartburn, for which the patient takes antacids and over-the-counter H2 blockers. There has been 10-lb weight loss in this past year. Social History: Negative Family History: Negative Physical Exam: This is pertinent for the patient being in mild-to-moderate distress. Hemodynamically, the patient is now stable after being in the hospital for the past 2 days with IV fluids. Abdominal exam is normal. Because of the nausea and vomiting, the patient underwent an upper endoscopy. This revealed severe ulcer esophagitis. There was a large volume of secretions within the stomach. The pH of this fluid was 1.0. There were multiple ulcers present in the duodenum, including the descending portion of the duodenum. Laboratory: Tests on arrival include Hgb 10 and hematocrit 30; transaminases were normal. Electrolytes were normal. Fasting serum gastrin was 5,000. ELISA H. pylori antibody was negative. An abdominal CT scan was negative for any tumor or mass within the abdominal cavity or liver. Based on these findings, which of the following would you recommend next? A) Start infusion with octreotide. B) Vagotomy and antrectomy. C) Chemotherapy to include 5-fluorouracil. D) Exploratory laparotomy. E) Endoscopic ultrasound.

Answer E. Endoscopic ultrasound. Explanation This patient has Zollinger-Ellison syndrome. Diarrhea is not uncommon and in fact occurs in 25% of cases. We usually think of Zollinger-Ellison with complicated peptic ulcer disease or recurrent ulcer disease, but diarrhea can indeed be the presenting symptom. This patient with severe esophagitis and multiple, atypical ulcers was a clue for the physician to order the gastrin level, which was quite high in this case. Once the diagnosis of Zollinger-Ellison is made, it is important to localize the site of the gastrinoma and determine if it is resectable. Often the tumor is so small that it defies diagnostic workup and cannot be found. In other cases, there are already metastases to the liver. In this described case, the CT scan did not locate the tumor, but the endoscopic ultrasound did identify a mass in the head of the pancreas. No lymph nodes were seen, and, therefore, this patient might be cured with a surgical resection of the tumor. The gastrinoma is a very slow-growing tumor. If the patient were a little older with other underlying medical problems, a strong case might be made for treating with a high-dose proton pump inhibitor alone—without any surgical resection. Most of these patients can be managed with medical therapy, although it does require quite aggressive dosing of the proton pump inhibitor.

A 54-year-old former rock and roll musician presents to your office because "I need my liver checked out." 10 years ago, he was told that he had hepatitis C. He received interferon treatment for the first 6 months but has not had follow-up since that time. He says that he feels fine except for occasional weakness. He denies ever having jaundice or confusion. He denies abnormal swelling or weight loss. Past Medical History: Otherwise negative Social History: The first line of the question says it all! He denies any ongoing alcohol intake or use of illicit drugs. Review of Systems: He denies any chest pain, shortness of breath, melena, change in bowel habits or abdominal pain. He has gained 10 lbs in the past year. Physical Exam: The patient is well nourished. There is no jaundice. There are no spider angiomas on physical examination. Lungs are clear and cardiac exam is normal. The liver is 2 cm below the right costal margin, and the spleen tip is palpable. There may be trace ascites present and there is no edema found. Laboratory: Tests reveal Hgb 12, WBC 7,200, platelet count 98,000. Bilirubin 2, AST 56, ALT 50, alkaline phosphatase 100, INR 1.1. An upper endoscopy is recommended, and this reveals large, grade IV esophageal varices. Some of the varices have red markings over them. There is no active bleeding. There are no definite varices in the stomach, although the stomach has an erythematous mosaic pattern. Based on the endoscopic findings, which of the following should you do next? A) No therapy for now, but repeat endoscopy in 6 months to assess varices. B) Start lactulose 30 cc bid. C) Endoscopic sclerotherapy with repeat exam until the varices are obliterated. D) Initiate spironolactone therapy 50 mg PO bid. E) Initiate propranolol and titrate the dose until the patient has a heart rate of < 60.

Answer E. Initiate propranolol and titrate the dose until the patient has a heart rate of < 60. Explanation This patient is at risk for bleeding from his esophageal varices. Although he has not bled, it would be appropriate to initiate beta-blocker therapy to reduce the risk of bleeding. Sclerotherapy is associated with complications, such as esophageal ulceration and stricture formation, and has not been proven to be of value for prophylaxis of gastrointestinal bleeding. If one of the choices had been variceal banding, this would have been a more difficult question because there have been studies that have shown a benefit to banding since it seems to have fewer complications than sclerotherapy. However, this is not universally accepted, and the standard of care at this time would still be beta-blocker therapy. Spironolactone might help with some of the patient's ascites but would not lessen the risk of bleeding. Lactulose would be indicated only for the treatment of encephalopathy.

A 55-year-old woman presents to the emergency department with the onset of moderately severe abdominal cramping associated with the passage of a maroon, bloody stool. This occurred suddenly earlier in the day without any prior symptoms. Since then she's had two additional passages of maroon stool, each time associated with significant cramps. Past Medical History: Significant for stable coronary artery disease. She has two stents in place but has good function and is active and free of angina. She takes a daily aspirin. Review of Systems: Otherwise negative Social History: She is active without any smoking or alcohol history. Family History: Negative for any significant gastrointestinal disease Physical Exam: The patient is in moderate distress, clutching her abdomen. She is afebrile with normal vital signs. The abdomen is slightly tender to palpation in the left lower quadrant with increased bowel sounds overall. The patient is started on IV fluids. Sigmoidoscopy is performed, which reveals a normal rectum. However, 25 cm from the anus in the sigmoid colon, there is an abrupt transition into colonic mucosa that is so edematous that a lumen cannot be identified. The mucosa is purple with what looks like submucosal hemorrhage. These changes are circumferential but begin very suddenly with an immediate transition to a normal mucosa distally. Based on this endoscopic appearance and clinical presentation, the best treatment at this time would be which of the following? A) Medicated cortisone enemas B) Surgical resection, subtotal colectomy C) Intravenous hydrocortisone D) Oral mesalamine E) Intravenous metronidazole

Answer E. Intravenous metronidazole Explanation By history, this patient has ischemic colitis. Many patients with ischemic colitis do not have any underlying risk factors. The association of lower GI bleeding with abdominal cramps is the key. Most patients will respond to bowel rest and IV hydration. However, when the involvement appears to be severe and circumferential, there probably is a role for an antibiotic with anaerobic coverage, like metronidazole. There is no suspicion in this case that this patient has inflammatory bowel disease; therefore, no role for mesalamine or hydrocortisone. At this time there is no role for surgical resection, which would only infrequently be appropriate in ischemic colitis since most cases will resolve with medical therapy.

A 40-year-old Irish-American man with a history of celiac disease presents with complaint of increasing fatigue for the past 3 months. He reports he has been compliant with his diet. He was not diagnosed until several years ago. His celiac disease resulted in growth retardation as a child. He occasionally suffers from dermatitis herpetiformis. He has a pale pallor to his skin and he looks "run down" to you. PAST MEDICAL HISTORY: As above MEDICATIONS: None SOCIAL HISTORY: Married, with 2 children Works as a leprechaun Has never smoked Drinks an occasional pint of beer FAMILY HISTORY: Sister has celiac sprue also; found to have HLA-B8 Mother and father healthy ROS: Severe fatigue Dyspnea on exertion with walking 1 block No chest pain No swelling in his legs No constipation PHYSICAL EXAMINATION: BP 120/60, P100, RR 18, Temp 99.0° F HEENT: PERRLA, EOMI, sclera non-icteric, very pale conjunctiva Neck: Supple; no bruits; no masses Heart: RRR with new II/VI systolic flow murmur Lungs: Clear to auscultation Abdomen: Mild epigastric tenderness to deep palpation Extremities: No cyanosis, clubbing, or edema GU: Heme-negative stool LABORATORY: CBC: WBC 8,500 with 60% polys, 30% lymphs Hemoglobin 8.5 mg/dL (previous hemoglobin 14 mg/dL) Rest of CBC pending Based on your extensive knowledge of celiac disease and putting together the physical exam findings with the laboratory, which of the following is the most likely diagnosis? A) Primary intestinal lymphoma B) Toxicity from wearing green leprechaun paint C) Celiac sprue exacerbation D) B12 deficiency anemia E) Iron deficiency anemia

Answer E. Iron deficiency anemia Explanation Remember that iron is almost completely absorbed in the duodenum. With celiac disease, this is one of the main sites of malabsorption. B12 deficiency is very common with tropical sprue, but not celiac sprue. Folate deficiency is also common but less so than iron deficiency. Celiac disease exacerbation would not be responsible for anemia of this degree with heme-negative stools and no history of blood in the stool. Primary intestinal lymphoma is a rare late complication of celiac sprue—and the key words here are rare and late—so this is not a concern for this 40-year-old leprechaun at this point.

An 18-year-old hip-hop artist presents with diarrhea for the past 2 weeks. She says that she noted the diarrhea began while she was performing in Mexico. You are the physician for the cruise line on which she is performing now. She tells you that she has had a low-grade temperature since returning from Mexico. She did not eat any fresh vegetables. She likes to eat beef jerky and prefers the "extra salty" version. She drank only bottled water and a soft drink for which she is a national spokesperson. She did have these drinks poured over ice, but she thought that the "frozen stuff would kill the cooties." She has not noted any blood in her stool. She has lost about 2 pounds in the last week. PAST MEDICAL HISTORY: Syphilis at the age of 15 Depression since age 16; on no medications at the moment SOCIAL HISTORY: Sexually active with multiple partners Smokes 1 ppd for the past 3 years Denies illicit drug use Denies use of alcohol FAMILY HISTORY: Mother with alcoholism; they are estranged at the moment Father left when she was 2 years of age Sister healthy, 20-year-old nun REVIEW OF SYSTEMS: Diarrhea is intermittent and she has crampy abdominal pain on occasion No rash No burning on urination No chills Diminished appetite PHYSICAL EXAMINATION: General: Pink hair with numerous piercings VS: Temp 100° F, BP 110/70, Pulse 95, RR 16 HEENT: PERRLA, EOMI Throat clear Heart: RRR with no murmurs, rubs, or gallops Lungs: CTA Abdomen: Hyperactive bowel sounds, nontender examination; no hepatosplenomegaly GU: Normal female genitalia, no tenderness on bi-manual palpation; no discharge noted Extremities: No cyanosis, clubbing, or edema Rectal: Heme-positive (slight) LABORATORY: Check for stool leukocytes: Positive Giardia specific antigen: Negative Stool culture: Salmonella enteritidis β-lactamase producing Based on this information, which of the following is the best treatment? A) Tetracycline 500 mg qid for 10 days B) Amoxicillin 500 mg tid for 10 days C) Ciprofloxacin 500 mg bid for 10 days D) Erythromycin 500 mg bid for 5 days E) No antibiotic therapy

Answer E. No antibiotic therapy Explanation With uncomplicated Salmonella gastroenteritis, antibiotic therapy is not indicated. If you treated her with antibiotics, you risk prolonging her shedding as well as increasing risk of resistance. Remember: Shigella you treat; Salmonella you generally don't. Exceptions are the very old, the very young, and the immunocompromised. We treat these special groups with antibiotics because the risk that the Salmonella may disseminate or cause more extensive problems is greater than the risk of prolonged shedding.

episodes of crampy abdominal pain with a sensation of feeling bloated 2 times a week associated with constipation for the last 8 years. She reports that her pain improves after defecation. She denies fever, rectal bleeding, or weight loss. She notes that she recently lost her job. She feels anxious because she already had to cancel one job interview due to abdominal pain and feels that this has caused her symptoms to worsen. She reports taking over-the-counter psyllium to try to improve her constipation without success. Her past medical history is otherwise unremarkable, and she currently takes no medications. Physical exam reveals a thin woman in no acute distress. Abdominal exam is notable for palpable bowel loop in the left lower quadrant with mild tenderness without rebound or guarding. The remainder of the physical exam is normal. Fecal occult blood testing is negative. Which of the following is the next best step in the management of this patient? A) Citalopram B) Senna C) Amitriptyline D) Dicyclomine E) Polyethylene glycol 3350

Answer E. Polyethylene glycol 3350 Explanation The patient in this scenario has a history of irritable bowel syndrome (IBS). IBS is classified as diarrhea predominant, constipation predominant, mixed diarrhea-constipation, and pain predominant. The treatment of IBS depends on the type of IBS. The majority of pharmacotherapy is targeted toward symptomatic relief. The patient in this vignette has constipation predominant IBS. Amitriptyline is frequently used in the treatment of IBS but is targeted toward the patient's pain and would be used for pain predominant IBS. The same is true of dicyclomine, an antispasmodic that has been shown to improve pain but not affect constipation or diarrhea. Citalopram is an SSRI used in the treatment of depression and anxiety. While it may be an effective therapy for her anxiety, it would not be considered as the first-line treatment for her IBS and would not resolve the underlying source of her anxiety—her IBS. Of the choices given, two are proven effective therapies for constipation: polyethylene glycol 3350 and senna. Polyethylene glycol is an osmotic laxative while senna is a stimulant laxative. Of these two choices, polyethylene glycol is the superior choice due to lower incidence of adverse effects and absence of tachyphylaxis that can lead to a further decrease in GI motility with prolonged use of senna

A 48-year-old Caucasian male with a history of Barrett esophagus presents for follow-up. He has been doing fairly well on omeprazole 20 mg daily, which seems to reduce his GERD symptoms to a minimum. Additionally, he stopped smoking several years ago and has lost about 20 pounds in the last 4 years. Today, he is returning for the pathology report from his last endoscopy, performed last week. Clinically, he has been doing well. LABORATORY: Pathology report: High-grade dysplasia confirmed by 2 pathologists. Now that he has confirmed high-grade dysplasia, which of the following is one of the currently recommended treatment options? A) Perform EGD monthly and watch for changes. B) Increase endoscopy frequency to every 6 months and intervene in 1 year if changes still exist. C) Increase dose of omeprazole to 40 mg and repeat EGD in 1 month; if no change, then recommend surgery or ablation. D) Apply low-grade beam radiation to the affected area. E) Refer for surgery.

Answer E. Refer for surgery. Explanation Now that he has high-grade dysplasia, he has 2 options according to the latest guidelines on Barrett esophagus published in 2008. The first option is standard resectional surgery, and the second option is to repeat EGD within 3 months and then follow 3-month EGD surveillance until intervention is needed. At this stage, decreasing gastric acid concentrations will not reverse the current damage; also, a 1-month time interval will not be useful in looking for changes. Surveillance every 6 months is too long a period of time in between. Monthly EGD is not cost-effective and very impractical. Finally, application of low-grade beam radiation to the affected area is not indicated and would harm the patient.

50-year-old man presents for follow-up of a gastric ulcer diagnosed 8 weeks ago. It was a relatively large, non-bleeding ulcer located in the duodenum. Helicobacter pylori was found in the biopsy specimen, as well as with a rapid urease test on the clinical specimen taken at biopsy. He took 14 days of a prepackaged triple-therapy agent (Prevpac®—consists of 30 mg lansoprazole, 1 g amoxicillin, and 500 mg clarithromycin). He was adherent with his medication regimen and reports no problem taking the medication correctly. He returns for follow-up 6 weeks after the completion of therapy. PAST MEDICAL HISTORY: No prior episodes of ulcer disease HTN for 20 years; currently on propranolol 20 mg q day SOCIAL HISTORY: Quit smoking 6 weeks ago; before then, 1 pack/day for 30 years No alcohol in last 6 months Works in a staple factory; packages staples FAMILY HISTORY: Noncontributory ROS: All symptoms now resolved No dyspepsia No chest discomfort No epigastric pain No difficulty with nocturnal pain No difficulty with pain after eating PHYSICAL EXAMINATION: VSS stable and documented correctly for Medicaid rules HEENT: No problems noted; non-icteric Heart: RRR without murmurs, rubs, or gallops Lungs: CTA Abdomen: Bowel sounds present in all 4 quadrants, no hepatosplenomegaly, no tenderness elicited on deep palpation of epigastric area Extremities: No cyanosis, clubbing, or edema Rectal: Heme-negative Which of the following tests should he be scheduled for now as follow-up? A) A urea breath test is indicated at this point to determine cure. B) It is too early to test for cure; you must wait until 3 months after therapy is complete. C) None; patients who are treated with triple therapy for H. pylori do not require follow-up assessment for cure because known cure rates with current regimens are nearly 95%. D) Repeat endoscopy is indicated because the other non-invasive tests are likely to still be false-positive this early after therapy is completed. E) Convalescent serology for H. pylori to look for 4-fold fall in titers.

Answer A. A urea breath test is indicated at this point to determine cure. Explanation About 75% of patients presumed to have uncomplicated peptic ulcer disease due to H. pylori infection are cured after 1 course of therapy. Treatment failure in the remainder of patients usually means that the ulcer will recur. Also, if untreated, these patients are at increased risk of complication such as gastrointestinal hemorrhage. The urea breath test is the best test to assess cure. A stool antigen test can also be used if the urea breath test is unavailable. Repeating EGD is indicated only in certain instances: 1) persistent symptoms after 1-2 courses of therapy, 2) patients with gastric ulcers, or 3) suspicion of gastric cancer. Cure confirmation can be done at 4-6 weeks after completing therapy. Patients cannot be on proton-pump inhibitors during breath urea testing because it interferes with the testing. Serologic testing is not effective for follow-up testing and also is no longer recommended for initial diagnosis.

A 43-year-old man presents to your clinic with concerns over long-term problems with diarrhea. He has experienced frequent loose stools for nearly two years. Of more concern to the patient is the fact that the stools are now associated with bloating and episodes of explosive diarrhea, and the associated smell is becoming almost unbearable. Other than the abdominal complaints, he has experienced some intermittent joint aching that seems to move from joint to joint and responds to occasional NSAIDs. His abdominal exam reveals some abdominal distention but no marked tenderness and no abnormal masses. Initial stool studies are remarkable for large quantities of fecal fat and a moderate amount of fecal leukocytes. Stool culture and examination for ova and parasites are negative. Along with additional laboratory studies, HIV testing is also negative. Subsequent evaluation included small bowel biopsy, which demonstrated PAS+ macrophages. Appropriate therapy for this condition would be: A) Antibiotic therapy B) Strict avoidance of gluten-containing foods C) Therapy with high-dose oral steroids D) Pancreatic supplements and avoidance of long-chain fats

Answer A. Antibiotic therapy Explanation Many causes exist for a malabsorption problem, and many present with diarrhea, steatorrhea, and abdominal distress. Avoidance of gluten is important in the treatment of nontropical sprue (celiac sprue). Reduction of long-chain fatty acids can reduce protein loss with intestinal lymphangiectasia. High-dose steroids are often used in the treatment of conditions such as inflammatory bowel disease. Whipple disease is suggested by PAS+ laden macrophages in tissue. Whipple disease is caused by the small gram+ bacillus, Tropheryma whipplei . It can involve the bowel, the eye, and the central nervous system as well as causing fever and migratory joint complaints. Treatment is initiated with IV ceftriaxone, and then the patient is placed on trimethoprim/sulfamethoxazole for a year

Four people arrive at the Emergency Department, all within 30 minutes of each other, presenting with similar symptoms. They all describe the sudden onset of nausea and severe vomiting. Shortly thereafter, all of them developed profuse diarrhea and now all complain of severe weakness. All 4 people had been together that afternoon—because they all work for a company in Colorado Springs that provides study materials for doctors taking board exams. The company was celebrating the summer with a picnic. There were a variety of different foods that were brought by different people, including deviled eggs, ham sandwiches, sashimi, barbecue chicken, hamburgers on the grill as well as raspberries and melon balls. None of the people involved remembers eating any other items. All had been swimming in a creek, and 2 admit to possibly ingesting some of the creek water. The nausea started almost exactly 4 hours after the picnic. On presentation to the Emergency Department, the person complaining of the most profound weakness has a blood pressure of 80/40 with a heart rate of 140 and decreased skin turgor. Temperature is 97° F. Abdomen is soft and nontender. IV fluids have been started. Which of the following is true? A) Antibiotic therapy is not indicated. B) This is likely E. coli O157:H7 related to undercooked hamburger. C) This is likely giardiasis from drinking the creek water in Colorado. D) Knowing the strange habits of the people who work for this company, exposure to iguanas is likely and Salmonella is a concern. E) This is probably Salmonella related to undercooking of poultry

Answer A. Antibiotic therapy is not indicated. Explanation These patients have the typical description of Staphylococcus aureus food poisoning. Patients usually present 4-6 hours after ingestion of the food with nausea, vomiting, and diarrhea. These symptoms do not last long, rarely more than 12 hours, but it is not uncommon to have severe dehydration. The S. aureus could have been obtained in the ham or even the deviled eggs. Although undercooked poultry can be a source of Salmonella, the incubation period is much longer than what is seen here. Likewise, in E. coli O157:H7, the incubation period is longer as well (3-4 days). Giardiasis can occur after ingesting Rocky Mountain water, but the symptoms are too severe for that to be the cause in this case, and the incubation period is 1-2 weeks. The treatment of S. aureus food poisoning revolves around supportive intravenous fluid. This is an ingested toxin rather than any active bacteria, so there is no value in giving antibiotics.

All of the following are indications for colonoscopy except: A) Bright red blood on toilet paper in an 18-year-old college student B) Gross lower gastrointestinal bleeding in a 30-year-old man C) Iron deficiency anemia in a 45-year-old man D) Hemoccult positive screen in a 60-year-old woman E) Streptococcus bovis bacteremia in a 60-year-old woman

Answer A. Bright red blood on toilet paper in an 18-year-old college student Explanation The 18-year-old likely has hemorrhoids. Everyone else needs colonoscopy. This is a break from all of those long-winded cases. Of course, if you were unlucky enough to get this question as your first question, then you have had your "easy" question and life is uphill from here. However, take heart—life might not be fair but MedStudy is (usually).

A 45-year-old African-American woman presents for her routine job screening physical examination. She is applying to be a security guard at her local airport. She is very healthy and works out at her local gym on a regular basis. PAST MEDICAL HISTORY: Negative except for hospitalizations for birthing her 2 children, which required C-sections On no medications SOCIAL HISTORY: Lives with her husband and 2 sons, ages 15 and 13 Has never smoked Drinks Mai Tais on a regular basis; 1 q day FAMILY HISTORY: Father diagnosed at age 48 with colon cancer; died 2 years later Mother 70, healthy Brother 42, obese REVIEW OF SYSTEMS: No headaches No sore throat No eye changes No muscle aches or pains No chest pain No cough No diarrhea No constipation No rashes Poor dentition—has been to dentist multiple times in last few months for cavities PHYSICAL EXAMINATION: Well-developed, well-nourished woman in no distress BP 110/70, P 65, RR 14, Temp 98.6° F HEENT: PERRLA, EOMI TMs clear Throat clear; poor dentition noted Neck: Supple, no masses Heart: RRR with murmurs, rubs, or gallops Lungs: CTA Abdomen: Auscultation normal; no hepatosplenomegaly Extremities: No cyanosis, clubbing, or edema GU: Normal female external genitalia; pelvic examination: Normal findings; non-tender exam Rectal: Results pending Which of the following would be the most appropriate colorectal cancer screening for her? A) Colonoscopy every 5 years beginning now. B) Begin annual fecal occult blood testing and flexible sigmoidoscopy every 5 years starting at age 60. C) Colonoscopy every 5 years beginning at age 50. D) Begin annual fecal occult blood testing and flexible sigmoidoscopy every 5 years starting at age 50. E) Colonoscopy every 10 years beginning now.

Answer A. Colonoscopy every 5 years beginning now. Explanation A positive family history is the most commonly identified factor that increases the risk of colorectal cancer. This gets a little confusing, so I will break it down in short paragraphs for you to "digest" more easily. 1. Persons with a single 1st degree relative with colorectal cancer that was diagnosed after age 60 experience a risk of colorectal cancer at age 40 that is equivalent to that of average-risk persons at age 50. Such persons should begin colorectal screening at age 40 (options for this: 1) colonoscopy every 10 years, or 2) annual fecal occult blood test plus flexible sigmoidoscopy every 5 years). 2. People at particularly increased risk are those with 2 or more 1st degree relatives with colorectal cancer or a single 1st degree relative diagnosed with colorectal cancer under the age of 60 (like this patient's father). The relatives of these affected patients should begin screening at age 40, or 10 years younger than the youngest affected 1st degree relative, whichever comes first. So, based on her family history and her dad's diagnosis at age 48, she should have started screening at age 38. Screening for these patients should be by colonoscopy at 3-5 year intervals depending on the strength of the family history.

A 25-year-old woman whom you've been following for the past 4 years for ulcerative colitis comes to the office with the joyful news that she and her husband are expecting their first child. She's 8 weeks pregnant at this time. She has had ulcerative colitis since the age of 20. A past colonoscopy demonstrated this is distal disease, or proctosigmoiditis. She has had approximately 2 or 3 flares of disease activity each year, each time being manifest as frequent bloody stools with mucus and tenesmus. During those times she has responded to oral prednisone added to her regimen of mesalamine. Previously she was on 1.6 g/day of mesalamine. Over the past 6 months this was increased to 3.2 gm of mesalamine/day, and this is seen to benefit her with fewer flares of the colitis. Past Medical History: Otherwise unremarkable Review of Systems: Negative for joint pain, skin lesions, mouth lesions or painful eye lesions Physical Exam: Normal For this patient with ulcerative colitis and new pregnancy, which of the following would be the most appropriate treatment? A) Continue mesalamine in its current dose. B) Continue mesalamine in the same dose and add 20 mg prednisone/day. C) Discontinue the mesalamine and initiate low dose prednisone 10 mg/day. D) Schedule colonoscopy to determine the disease activity prior to any changes in therapy. E) Add azathioprine to the mesalamine with the hopes of preventing any flares during pregnancy.

Answer A. Continue mesalamine in its current dose. Explanation It's difficult to predict what will happen to this young woman during her pregnancy. The colitis could flare, requiring prednisone use; however, prednisone is well tolerated during pregnancy. The patient may be quite well during her pregnancy without any flares and then have an exacerbation of the disease in the immediate postpartum. However, the most appropriate treatment at this time is to continue the mesalamine dose that seems to be providing a beneficial effect up until the time of her pregnancy. Although this has not been directly tested during pregnancy, it is felt to be safe. It is certainly safer than stopping the drug and exposing her to disease exacerbation, which could be serious. There is no reason to schedule a colonoscopy. One can usually determine the disease activity based on her symptomatology.

A 19-year-old woman presents with daily acid reflux symptoms and intermittent dysphagia to solids. Since onset of symptoms 3 months ago, she has lost 4 pounds. She also has sporadic rectal bleeding. Her past medical history is negative. Her father had stomach cancer at age 42. What would you recommend? A) EGD B) Reassurance C) Barium esophagram D) PPI therapy for 6-8 weeks

Answer A. EGD Explanation This patient likely has uncomplicated reflux disease, but her family history of GI cancer, blood in her stool, and dysphagia are all "alarm" symptoms that merit workup. The patient needs more than just medication or reassurance at this point, though the workup is still likely to be negative. EGD is the correct answer. Barium esophagram could be considered, but her dysphagia to only solids is concerning for a stricture, and limiting her diagnostic/therapeutic evaluation to EGD with possible dilation would probably be the best course of action.

You are seeing a 48-year-old Puerto Rican man with a history of colonoscopic excision of a 3-cm sessile polyp earlier today. He is otherwise healthy and has not had any health issues come up in the past 3 years that you have been following him. He has a history of mild hypertension controlled with diet but otherwise has not been into the office except for an occasional upper respiratory infection. PAST MEDICAL HISTORY: As above SOCIAL HISTORY: Works as an attorney Smokes 2 ppd of cigarettes Drinks 3-4 beers nightly FAMILY HISTORY: Father, healthy Mother, 75, lives in Puerto Rico, healthy No siblings REVIEW OF SYSTEMS: No headache No sore throat No fevers No night sweats No chest discomfort Chronic smoker-type cough; worse in the morning on awakening No weight loss PHYSICAL EXAMINATION: Had complete physical 2 months ago; nothing has changed since that visit LABORATORY: Pathology shows resection of a large 3-cm sessile polyp with villous tissue. The pathologist reports that she is reasonably sure the polyp was completely removed—the edges are distinct; no malignancy was found in the biopsy taken. Based on these findings, which of the following should be your recommendation? A) Schedule for routine colonoscopy in 10 years. B) Schedule for routine colonoscopy in 5 years. C) Repeat colonoscopy in 1 year. D) Refer to surgeon for surgical resection of affected area. E) Repeat colonoscopy in 3-6 months to be sure that resection was complete.

Answer E. Repeat colonoscopy in 3-6 months to be sure that resection was complete. Explanation Large sessile polyps (> 2 cm) usually contain (as his did) villous tissue with a high malignant potential and tend to recur locally after resection. Frequently, these lesions cannot be completely or safely excised during colonoscopy, and the patient should be referred for primary surgical resection. However, in this case, complete excision was possible or thought possible, and it was done at time of colonoscopy. If a patient has had a successful colonoscopic excision of a large sessile polyp, then he/she should undergo follow-up colonoscopy 3-6 months later to determine whether the resection was complete. If at this point a residual polyp is present, it should be removed and the completeness of resection documented again within 3-6 months. If 2 or 3 attempts at removing a polyp are not successful, surgical referral is indicated.

Which of the following is the least common GI location for carcinoid tumor? A) Appendix B) Rectum C) Ileum D) Colon E) Stomach

Answer E. Stomach Explanation Carcinoid rarely affects the stomach.

A 55-year-old male complains of upper abdominal discomfort and fullness after meals. He also has frequent belching. These symptoms have been present for several months. He gives no prior history of similar symptoms. The symptoms have been persistent despite the use of his wife's prescription of famotidine 20 mg twice a day for the last 8 weeks. Past Medical History: Significant for hypertension. His only medication is benazepril. He denies any aspirin or NSAID use. Review of Systems: Negative for weight loss Physical Examination: Reveals a benign abdomen Upper GI has been performed and shows a gastric ulcer of benign appearance. Which of the following is the most appropriate next step? A) EGD now. B) Continue famotidine for 8 more weeks. C) Empiric omeprazole and re-evaluate in 8 weeks. D) CT scan.

Answer A. EGD now. Explanation Although the upper GI suggests this is a benign ulcer, the features of the benign ulcer are not always accurate to differentiate benign from cancer. This may actually be difficult to define at EGD as well. The fact that this ulcer is present despite H2 blocker therapy is worrisome. Just from the history alone, however, the onset of dyspeptic symptoms at this age is enough to do an EGD from the outset and not waste the barium or the radiologist's time with an upper GI series.

A 52-year-old female originally presented to the emergency department with severe acute abdominal pain. This pain was unlike any pain she had ever experienced in the past. It radiated to the back and was associated with nausea and vomiting. The pain started 8 hours earlier and had been constant since that time. PAST MEDICAL HISTORY: Significant for hyperlipidemia; the only medicine is atorvastatin FAMILY HISTORY: Positive for CAD SOCIAL HISTORY: Denies any alcohol use; does not smoke cigarettes REVIEW OF SYSTEMS: She denies any change in her bowel habits, chest pain, or SOB PHYSICAL EXAMINATION: She is in moderate distress; HR 110, BP 110/60 Skin: Turgor is normal HEENT: Unremarkable Chest: Clear to auscultation Cardiovascular: Normal, except for the tachycardia Abdominal exam: Tender in the epigastrium to mild palpation; there is no rebound or guarding; there is no referred pain on exam; stool is heme-negative Extremities: Normal LABS: Amylase 1,242, lipase 865, bili 1.9, AST 86, ALT 92, alk phos 245, Hgb 15, WBC 13,000 An ultrasound was done on admission, which revealed a gallbladder with multiple filling defects consistent with stones. The pancreas is edematous. The common bile duct was identified and was of normal diameter. On the 2nd day of admission, the amylase increased to 1450, and the bilirubin climbed to 3.5. However, at the time, the patient felt much better. She was receiving IV meperidine, but felt she didn't need this anymore. It is now the 5th day and her amylase has been rechecked and found to be completely normal. Her abdomen is soft and nontender. She is not jaundiced and is now tolerating a liquid diet. Which of the following studies would you recommend? A) ERCP now, since there is a high likelihood of stones present in the CBD. B) Open cholecystectomy with common bile duct exploration now before discharge from the hospital. C) Discharge from the hospital now and then elective cholecystectomy in 2 weeks. D) Percutaneous transhepatic cholangiogram.

Answer A. ERCP now, since there is a high likelihood of stones present in the CBD. Explanation This woman has had a case of pancreatitis likely secondary to a passed gallstone. Although the ultrasound did not show any stone or stones in the common duct or dilatation in the common duct, there are several factors here that would predict a moderate possibility of persistent stones in the duct. This would be the fact that her laboratories increased on the day after admission and that her bilirubin went as high as 3.5. Therefore, with this moderate possibility of retained stones in the CBD, an ERCP is reasonable. ERCP does not necessarily have to be done before cholecystectomy. Some would say do a laparoscopic cholecystectomy with an intraoperative cholangiogram, and then proceed with an ERCP after surgery if the cholangiogram is positive for stones. However, this is a case where it is probably appropriate to do the study preoperatively. There is no value in sending the patient home before coming back to have surgery in this case, since her abdomen is completely benign. Likewise, there is no reason why she can't have a laparoscopic cholecystectomy, and there is no need for the increased morbidity associated with the open procedure with common duct exploration. Although percutaneous cholangiogram will clearly identify stones in the common duct, it does not have the same therapeutic potential of being able to remove the stones readily that an ERCP does.

A 32-year-old woman presents to your office with a complaint of abdominal pain. She describes generalized pain that occurs 4-5 times per month since she was a teenager. She notes that her stools often become loose after she notices the pain, but that it improves after she defecates. She reports she occasionally sees blood in her stool with intense straining during a bowel movement. She denies fever, nausea, vomiting, or weight loss. Her past medical history is significant for generalized anxiety for which she takes citalopram. Physical exam is notable for mild tenderness to palpation in the LLQ without rebound or guarding. Digital rectal exam demonstrates normal tone without palpable mass. The remainder of the exam is normal. Which of the following is the next best step in the management of this patient? A) Fecal occult blood testing. B) Begin therapy with amitriptyline. C) CT of the abdomen. D) Tissue transglutaminase antibody testing. E) Begin therapy with loperamide.

Answer A. Fecal occult blood testing. Explanation This case represents a history that is initially consistent with irritable bowel syndrome (IBS). She has abdominal pain that occurs > 3 times per month for at least the last 3 months that improves with defecation and is associated with a change in stool caliber. However, blood in the stool is an alarm symptom that does not fit the diagnosis and raises concern for another diagnosis such as inflammatory bowel disease or neoplasm. The next most appropriate test would be to confirm the presence of blood in the stool with fecal occult blood testing. Once confirmed, additional evaluation of the patient's pain would be warranted. Tissue transglutaminase would be useful in the evaluation of celiac sprue. This would be likely in the absence of warning symptoms and is considered in the differential diagnosis of IBS-like symptoms. A colonoscopy would be warranted if you had other data to suggest a diagnosis of inflammatory bowel disease. A CT of the abdomen at this point in her evaluation would be premature. Beginning therapy for diarrhea-predominant IBS with loperamide or pain-predominant IBS with amitriptyline would not be recommended given the presence of an alarm symptom that warrants additional investigation.

A 60-year-old woman presents with a sudden onset of cramping abdominal pain. This pain is in the lower abdomen and was associated with passing several maroon stools. The pain is severe, but intermittent. All of these symptoms started 4 hours ago. She has never had symptoms like these before. Past Medical History: Significant for CAD. She has had several stents placed. She is on daily aspirin. She quit smoking 20 years ago and does not drink alcohol. Review of Systems: She has always had normal bowel movements in the past. She denies any syncope, chest pain, or recent weight loss. Physical Examination: She is in mild distress, but mostly resting comfortably. Her HR is 100 and BP is 120/60. Skin: Normal HEENT: Unremarkable and sclera are anicteric. There is no lymphadenopathy. Cardiac Exam: Normal Chest: Clear Abdomen: She demonstrates mild tenderness in the LLQ. There is no hepatomegaly. The abdomen is not distended, and bowel sounds are normal. There is no mass present. Rectal Exam: Maroon stool is present. Laboratory: Hgb 11.6, WBC 7,200, PT 12, INR 1 What is the next appropriate test? A) Flexible sigmoidoscopy B) Upper GI small bowel series C) CT scan of abdomen and pelvis D) Mesenteric angiography

Answer A. Flexible sigmoidoscopy Explanation This woman has a classic presentation of ischemic colitis, which is rectal bleeding associated with abdominal cramps. This is in contrast to the most common cause of lower GI bleeding, which is diverticular bleeding. In diverticular bleeding, there are no significant cramps. Very rarely would angiography be indicated in ischemic colitis. Ischemic colitis occurs because of a transient low-flow state to the colon rather than any fixed obstruction or embolic event. It usually involves the sigmoid colon extending to or through the splenic flexure. Therefore, a simple flexible sigmoidoscopy may reveal a normal rectum and then show prominent edema and submucosal hemorrhage that begins abruptly in the sigmoid colon. CT scan could certainly demonstrate some thickening in the sigmoid colon, but it would not allow for the definitive diagnosis like a sigmoidoscopy would. There is no value here for any GI series. Treatment at this time would consist of intravenous fluid support to maintain good volume. Most cases will resolve with conservative management.

The father of a family calls the emergency department at midnight with a query for advice and gives the following scenario. The entire family, within a 1-hour time period, noted the onset of profuse vomiting. Each person had multiple episodes of retching. The persons involved include the husband, wife, and two teenage children. The husband and wife have started to manifest some diarrhea as well. No one has a fever, although each person does have some abdominal cramps. None has any underlying medical illnesses, nor have they experienced anything like this before. On further review, we find that both teenagers had eaten fast-food hamburgers and french fries for lunch. The entire family went to a Chinese food restaurant that evening. The wife mentions that she threw out some mayonnaise from the refrigerator that smelled bad the day before. Which of the following is the most likely cause of the described illness? A) Food poisoning with B. cereus B) Salmonella C) Food poisoning with Staphylococcus aureus D) Viral gastroenteritis E) E. coli O157:H7

Answer A. Food poisoning with B. cereus Explanation The key here is the Chinese food restaurant. There is a well-described syndrome of food poisoning with B. cereus that is related to fried rice. This is a toxin-mediated illness very similar in timing and symptomatology to S. aureus. The delay in symptom onset is usually 4-6 hours, with the symptom of vomiting followed by diarrhea. When one thinks of S. aureus food poisoning, one usually associates this with a picnic where food has been left outside, including sandwiches that have mayonnaise or other sweets. Although E. coli O157:H7 can be associated with fast-food hamburgers, the timing here is too fast and all the members of the family did not eat the hamburgers (onset usually 3-4 days). There are no incriminating foods to suggest Salmonella. Viral gastroenteritis can present with similar symptoms, but one would not expect everyone to be getting sick at the same time like they did in this case.

A 35-year-old male has a new-onset diarrhea, which started 3 days ago. Initially, it was watery stool but then, 2 days ago, he started passing blood as well. He has had associated abdominal cramps and nausea. He has vomited once as well. He was previously normal and has never had any significant gastrointestinal symptoms in the past. Past Medical History: Unremarkable Social History: He denies any foreign travel. He has no history of recent antibiotics. He has a hectic schedule and frequently eats fast-food hamburgers, almost on a daily basis. Family History: Negative for inflammatory bowel disease or any other significant GI disease Review of Systems: Does admit to feeling weak. He denies any fever or chills. Physical Examination: HR 100, BP 100/60. Skin turgor is slightly decreased and mucous membranes appear dry. There is no lymphadenopathy. Pulmonary and cardiac exam: Normal Abdomen: Nondistended. Bowel sounds are hyperactive. No rushes or tinkles. He is slightly tender in the RLQ although without a palpable mass. Laboratory: Hgb 16, WBC 12,000. A number of stool studies have been drawn. Routine cultures, including Salmonella and Shigella, are negative. He is positive for E. coli O157:H7. Currently there are pending tests of the stool for Clostridium difficile toxin and ova, cysts, and parasites. Which of the following antibiotics would you recommend? A) No antibiotic treatment B) Amoxicillin-clavulanic acid (Augmentin®) 875 mg bid x 10 days C) Ciprofloxacin 500 mg bid x 7 days D) Metronidazole 500 mg tid x 10 days

Answer A. No antibiotic treatment Explanation This patient with his history of frequent fast-food hamburgers has come in contact with E. coli O157:H7, which was confirmed in the appropriate stool studies. His symptoms are also typical of that, as it often causes bloody diarrhea. This bacterium has the propensity for involving the right side of the colon, hence the tenderness in the RLQ. It also tends to cause nausea and vomiting more often than the other bacterial colitides. Even though the stool for C. diff toxin is not back yet, there is no reason to suspect this, since he has not been on antibiotics. Amebiasis can cause bloody diarrhea, but with the history not showing any foreign travel, this would be very unlikely. Most believe that the most appropriate treatment of E. coli O157:H7 is merely supportive therapy, including appropriate IV fluid hydration. Antibiotics may increase the risk of developing complications, such as hemolytic uremic syndrome, and therefore should not be employed once the diagnosis is known or confirmed.

A 25-year-old male presents with a history of deep burning epigastric pain for the past week. The pain is worse 1 to 3 hours after eating and is relieved by ingestion of antacids or food. The pain has also awakened him from sleep at night on occasion. He has no anorexia, dysphagia, vomiting, or weight loss. He smokes a pack of cigarettes daily. He denies use of NSAIDs. PAST MEDICAL HISTORY: Gonorrhea at age 18 Syphilis at age 19 SOCIAL HISTORY: Waiter at local Mexican restaurant Smoking history as above; started smoking at age 15 No illicit drug use since age 19 (cocaine and marijuana); no IV drug history Alcohol: Abstinent since age 19; attends regular AA meetings Sex: Uses condoms every time; no sexual encounters in 6 months FAMILY HISTORY: Father 55 with diabetes Mother 60 with hypertension, mild CVA last year Sister 23 with no health problems REVIEW OF SYSTEMS: No fever or chills No tachycardia Frequent dyspepsia Frequent burping No heartburn while lying flat No regurgitation No other pain besides the deep epigastric pain No nausea or vomiting No GU symptoms PHYSICAL EXAMINATION: BP 130/80, P 75, RR 14, Temp 99° F, Ht 6' 1", Wt 220 HEENT: PERRLA, EOMI TMs clear Throat clear Neck: Supple; no thyromegaly Heart: RRR with murmurs, rubs, or gallops Lungs: CTA Abdomen: Bowel sounds present in all 4 quadrants, slightly hyperactive; no hepatomegaly; spleen tip non-palpable; mid-epigastric tenderness to deep palpation; no rebound Extremities: No cyanosis, clubbing, or edema Skin: No rash, except mild acne Rectal: External hemorrhoids, heme negative GU: No lesions noted; normal male genitalia LABORATORY: CBC: WBC 8,500 with 60% polys, 40% lymphocytes; hemoglobin 15.5 mg/dL, hematocrit is 55% MCV 90 Electrolytes: Normal Based on your history, physical, and laboratory tests, which of the following is the next step in evaluation of this patient? A) Noninvasive testing for Helicobacter pylori. B) Esophagogastroduodenoscopy (EGD). C) Treat empirically with H-2 blockers; if no improvement in 2 weeks then test for Helicobacter pylori. Barium swallow. D) Esophagogastroduodenoscopy (EGD) with biopsy, because this is the only definitive method to determine if Helicobacter pylori is the etiology of his symptoms.

Answer A. Noninvasive testing for Helicobacter pylori. Explanation In a healthy young person (defined by the American Society of Gastroenterology as being anyone under the age of 45—complain to them if you are over 45!) with no "alarm" symptoms/signs, noninvasive Helicobacter pylori testing (without EGD) is indicated because the risk of gastric carcinoma is low. Indications for early EGD are anorexia, dysphagia, gastrointestinal bleeding (gross or occult), new-onset symptoms in persons > 45 years old, presence of a mass, unexplained anemia, unexplained weight loss, or severe vomiting. Noninvasive testing in symptomatic patients and treatment in those with positive results are cost-effective. Remember: Barium swallow is a test for dysphagia, not dyspepsia. EGD with biopsy is the most specific test for diagnosis of H. pylori-induced peptic ulcer disease, but is rarely indicated as the initial diagnostic test in uncomplicated ulcer disease in a young healthy person. On the biopsy specimen, you can actually look for histological evidence of the little critter (H. pylori) or do the rapid urease test on the specimen. Both are very specific for infection if present. Remember that if this were an older person (> 45 years), you would proceed to EGD with biopsy of suspicious lesions. So remember: under 45 ... no EGD; over 45 ... EGD for peptic ulcer workup. Finally, empirical H2 blocker therapy alone would be ineffective for H. pylori infection.

40-year-old woman has suffered with chronic heartburn and diarrhea for 2 years. She takes over-the-counter omeprazole, which gives some relief of the heartburn. She also complains of chronic diarrhea, with 5 to 6 loose bowel movements a day and some epigastric discomfort. EGD is done and shows grade III esophagitis. Multiple shallow ulcers are seen in the duodenal bulb and second portion of the duodenum. You recommend which of the following? A) Order fasting serum gastrin after stopping the omeprazole for 7 days. B) Order H pylori ELISA. C) Continue omeprazole, but add metoclopramide before meals. D) Continue omeprazole, but add sucralfate qid.

Answer A. Order fasting serum gastrin after stopping the omeprazole for 7 days. Explanation Remember Zollinger-Ellison syndrome, especially if chronic diarrhea accompanies either bad esophagitis or peptic ulcer disease. Continuing omeprazole with sucralfate or metoclopramide is not going to be helpful, and she needs further evaluation. H. pylori testing is reasonable, but the "illness script" here of diarrhea and persistent heartburn on omeprazole should make you think of Z-E syndrome. During the long-term treatment of a 26-year-old male with ulcerative colitis, the patient requires intermittent pulses of glucocorticosteroids to gain symptomatic control.

A 46-year-old male patient presents to the clinic with complaints of repeated vomiting. Of greatest concern to the patient is the presence of blood in the emesis. Which of the following is most specific for peptic ulcer disease as the source for the vomited blood? A) Specks of blood with black particulate emesis B) Large volumes of regurgitated bright blood C) Bright red blood following an emesis episode D) Mucoid emesis with streaks of blood

Answer A. Specks of blood with black particulate emesis Explanation Blood in the gastrointestinal tract can originate anywhere from the nose and mouth to the anal opening. Sources in the upper GI system can lead to blood in the emesis with vomiting. Though no pattern is pathognomonic for the source of an upper GI bleed, some characteristics are typical. Vomiting mucous material with streaks of blood suggests an upper airway irritation with swallowed secretions. Large volumes of regurgitated blood suggest esophageal varices. Hematemesis that follows an episode of vomiting suggests a Mallory-Weiss tear. Specks of blood with emesis that is "coffee ground" (black particulate) in appearance is consistent with bleeding from an upper GI source that has been in contact with acidic secretions. Of the list provided, this is the choice that is most specific for bleeding from a peptic ulcer.

A 40-year-old male has had ulcerative colitis since age 25. He has been having one or two flares per year. He moved to your city 5 years ago and has not established himself with a doctor. He has been using the prednisone prescription from his last physician for his rare flares. He was told at his last colonoscopy 5 years ago that his colitis involved the entire colon. He currently has normal bowel movements and denies any blood or mucus. Past Medical History: Is otherwise healthy Social History: He smokes a cigar nightly and works as a stockbroker. Review of Systems: Negative for joint pain or mouth ulcers Physical Examination: Completely normal Which of the following do you recommend? A) Start sulfasalazine 2 g/day. B) Start azathioprine for steroid-sparing effects. C) Quit smoking because it will help his colitis. D) Colectomy because of the cancer risk after 15 years of pancolitis.

Answer A. Start sulfasalazine 2 g/day. Explanation This patient needs careful screening and surveillance for colon cancer but not a prophylactic colectomy. He should have total colonoscopy with extensive biopsies for surveillance done on an every-other-year basis at this time. It is rare for ulcerative colitis patients to be smokers. If this patient smoked cigarettes, his disease might actually get worse if he were to stop smoking. However, smoking a cigar each night may not have the same effect. Currently, he has mild symptomatology and rarely active disease. This would be a good situation to start something like sulfasalazine or mesalamine rather than become more aggressive with azathioprine.

A 45-year-old female is seen in your clinic complaining of swallowing difficulty and vomiting. The patient has complaints of intermittent regurgitation for several years but has recently been concerned with the frequency of episodes. When she vomits, the emesis consists of nonbilious fluids and particles of undigested food. She denies any blood or significant pain. She recently became more concerned when the emesis contained food residua from a meal eaten several days previously, despite having eaten several times since then. The physical exam is most remarkable for significant halitosis. Her abdominal exam is unremarkable. Screening CBC and electrolytes are within normal limits. Which of the following is the most likely diagnosis? A) Zenker diverticulum B) Achalasia C) Mallory-Weiss tear D) Plummer-Vinson syndrome

Answer A. Zenker diverticulum Explanation This patient's presentation is most consistent with a Zenker diverticulum. Vomiting undigested food several days after consumption indicates an area of sequestration as in a Zenker diverticulum. Achalasia can demonstrate regurgitation of undigested food but does not account for the intervening passage of subsequent food boluses. Plummer-Vinson syndrome includes iron deficient anemia, which is not noted in this case. Mallory-Weiss tears can accompany any vomiting episode but results in hematemesis and often includes esophageal pain, neither of which is present in this case.

A 25-year-old male presents to the office complaining of intermittent chest pain for the past year. This is a brief pain that lasts only 1-2 minutes in duration. It is unpredictable and not related to meals. It never wakes him. He says that it can radiate into the left chest, and he describes it as a sharp stabbing pain. There are no other factors in his past medical history. He does have a family history of premature coronary artery disease. In his social history, he admits to significant stress in his job as an IRS auditor. In review of systems, he denies any dysphagia or any typical reflux symptoms. Physical examination is normal. Stress echo: Normal EGD: Normal Esophageal motility: This demonstrated mostly normal peristalsis, but there were intermittent simultaneous contractions seen. Lower esophageal sphincter pressure was 40, but with complete relaxation. The amplitude in the esophageal body was 140 (normal 40-180). Which of the following is true? A) You should get a CT of the chest. B) 24-hour ambulatory pH probe may reveal abnormal reflux even without typical reflux symptoms. C) Additional cardiac tests are needed (e.g., cardiac catheterization). D) Nitrates are effective therapy and well tolerated for this condition. E) Empiric dilatation may help this pain.

Answer B. 24-hour ambulatory pH probe may reveal abnormal reflux even without typical reflux symptoms. Explanation By definition, this patient's motility shows diffuse esophageal spasm, which is characterized by intermittent simultaneous contractions amid normal peristalsis. The upper endoscopy in this case was a very low-yield procedure and probably added little to the management of this patient. However, a 24-hour ambulatory pH probe may reveal that some cases of diffuse esophageal spasm are due to reflux, even if the patient does not have typical reflux symptoms. In these cases, the chest pain will resolve with aggressive treatment of the reflux. Some cases of diffuse esophageal spasm will have dysphagia with the chest pain. However, in this case, since there is no dysphagia, there is no reason to expect the dilatation would help. Probably the most important aspect of treatment is reassurance of the patient. Nitrates are poorly tolerated on a long-term basis for this presentation. There may be a role for use of calcium-channel antagonists, anti-spasm medicines, or tricyclic antidepressants.

A 55-year-old male is in for an executive health check. He has no specific complaints and is not taking any medications. He doesn't have any family history of colon cancer. As part of the exam, you recommend a screening colonoscopy and this was performed. On this exam, a 1.2-cm polyp in the ascending colon was removed with snare technique. The pathology of the polyp showed a tubular adenoma with low-grade dysplasia. The patient now asks you when he should have a follow-up colonoscopy. Which of the following recommendations do you give the patient regarding follow-up colonoscopy? A) 1 year. B) 3 years. C) 10 years. D) No follow-up is needed. E) 5 years.

Answer B. 3 years. Explanation Having just 1 adenoma with low-grade dysplasia would normally mean followup in 5-10 years. However, because the adenoma was > 1 cm in size, this increases his risk, and therefore he should return in 3 years for his next colonoscopy. Note: Once polyps are found and removed, subsequent screening should always be done with full colonoscopy and not one of the other methods of screening (fecal occult testing, sigmoidoscopy, barium enema, etc.). Know the appropriate intervals for colonoscopy: 10 years: no polyp or hyperplastic polyp For all below, colonoscopy only choice! 5-10 years: 1 or 2 small tubular adenomas with low-grade dysplasia 3 years: 3-10 adenomas or 1 adenoma > 1 cm or Any villous or high-grade dysplasia < 3 years: > 10 adenomas on a single exam 2-6 months: Sessile adenomas that are removed piecemeal

A 32-year-old white female is referred to you for anemia. She has had Crohn disease for 15 years and has had one surgery, but she doesn't remember the details. She is currently on no medications. She has diarrhea, moving her bowels about 8 times per day. Once or twice a month she also has postprandial vomiting. Her Hct is 30%, and her MCV is 102. Her Crohn disease is likely which of the following? A) Inactive, in the terminal ileum B) Active, in the terminal ileum C) Inactive, gastroduodenal D) Active, gastroduodenal

Answer B. Active, in the terminal ileum Explanation The patient continues to have obstructive symptoms and likely has active disease. She also has macrocytic anemia which argues in favor of distal small bowel disease (B 12 deficiency).

A 30-year-old woman has long-standing Crohn disease. She had one prior surgery 3 years ago, at which time 24 inches of the ileum and cecum were removed. She has had no other surgeries and takes mesalamine 1.6 gm each day. There has been no evaluation in the past 3 years. She comes into the office with 2 months of intermittent RLQ pain and loose stools. She denies fever, nausea, or vomiting. Exam reveals tenderness in RLQ but without mass. CBC and LFTs are normal. Which of the following is the most likely diagnosis for this patient? A) Gallstones B) An exacerbation of Crohn disease C) Kidney stones D) Primary sclerosing cholangitis

Answer B. An exacerbation of Crohn disease Explanation Gallstones, kidney stones, and PSC can all be seen in Crohn disease. However, most patients—after a resection—will have a recurrence of the Crohn's, and the presentation of chronic diarrhea and abdominal pain in the RLQ would favor this. Gallstones and PSC you would expect RUQ pain and with kidney stones, flank pain.

An 85-year-old African-American woman presents to your office with complaints of lower abdominal pain. Her pain is crampy and bilateral but worse on the right side. The pain started approximately 24 hours ago. The location has not changed, but the intensity of the pain increased overnight. The pain is accompanied by nausea. She vomited 2 or 3 times the preceding day. She also reports diarrhea and chills during the past 24 hours. PAST MEDICAL HISTORY: Coronary artery disease, for which she had angioplasty 3 years ago; no angina symptoms since Diverticulosis without history of diverticulitis since 1989 She has well-controlled hypertension, gout, and chronic low back pain Past surgical history: hysterectomy/bilateral salpingo-oophorectomy in 1976 MEDICATIONS: Atenolol 50 mg q day Allopurinol Tramadol prn ECASA 81mg q day SOCIAL HISTORY: No alcohol consumption No cigarette smoking; but does chew tobacco She has no history of drug abuse FAMILY HISTORY: No living relatives; doesn't know her parents or siblings PHYSICAL EXAMINATION: Temp 101.3° F, P 128, BP 127/50, RR 18, oxygen saturation of 98% on room air; she is 5 feet tall and weighs 163 pounds; she appears to be in mild distress secondary to pain; she is alert and fully oriented HEENT: Her oral mucosa appears dry; PERRLA, EOMI Neck: No masses, no bruits Heart: RRR with tachycardia Lungs: Clear to auscultation Abdomen: Relatively diffuse tenderness over the lower abdomen, more severe over the right lower quadrant; the abdomen was soft without hepatomegaly or splenomegaly; there was no rebound or guarding; bowel sounds were hypoactive Rectal: Heme-negative stool Neurological examination was intact LABORATORY: Leukocyte count was 10.0 x 109/L with 84% neutrophils; hemoglobin concentration was 12.1 g/dL;platelet count was 181,000 Chemistries: Sodium 143 mEq/L, serum potassium 4.6 mEq/L, serum chloride 108 mEq/L, serum bicarbonate 21 mEq/L, blood urea nitrogen 44 mg/dL, serum creatinine 1.5 mg/dL, glucose 169 mg/dL Amylase 113 U/L, lipase 50 U/L, aspartate aminotransferase 44 U/L (14-36), alanine aminotransferase 59 U/L (9-52), alkaline phosphatase 68 U/L, and low-density lipoprotein 784 U/L (313-618) Urinalysis showed 1-2 white blood cells, +1 leukocyte esterase, +1 bacteria, and nitrite negative Electrocardiogram showed sinus tachycardia with a rate of 103 without ST segment or T wave changes Chest x-ray showed minimal bibasilar atelectasis Flat and upright abdomen x-ray was negative Ultrasonography of the right upper quadrant showed no signs of cholecystitis Computed tomography of the abdomen without intravenous contrast showed mild circumferential mural thickening of the terminal ileum, mild inflammation of mesenteric fat, the appendix not visualized, and no free or localized fluid Which of the following is the most likely diagnosis based on these findings? A) Colon carcinoma B) Appendicitis C) Viral gastroenteritis D) Crohn disease E) Diverticulitis

Answer B. Appendicitis Explanation Right lower quadrant pain with fever should always raise the suspicion for appendicitis. The typical presentation of appendicitis is initial peri-umbilical pain, which then localizes to the right lower quadrant. Fever and leukocytosis are also typical of appendicitis. The presentation of appendicitis in the elderly is often atypical. Pain can be poorly localized or even be absent. Fever may not be present. Leucocytosis is variable. Of patients older than 75 years of age, 80% present with atypical symptoms, as compared with 80% of patients younger than 65 years who present with typical symptoms. The rate of perforation is high in the elderly because of the delay in making a definite diagnosis of appendicitis. Appendicitis is still largely a clinical diagnosis. Computed tomography with colonic contrast has been reported to be 98% sensitive and 98% specific in experienced hands in diagnosing appendicitis. However, the sensitivity is variable depending on the radiologist, with sensitivity as low as 50%. The addition of intravenous contrast has been reported to increase sensitivity by about 25%. Crohn disease is least likely for several reasons. Crohn disease is more prevalent in Caucasians than in African-Americans or Asians. It typically presents at an earlier age, with peak incidence at 15 to 35 years of age. The presentation is usually subacute with recurrent episodes of right lower quadrant abdominal pain, low-grade fever, diarrhea, and possibly a right lower quadrant mass. Acute ileitis, however, may have an abrupt onset with fever, leucocytosis, and abdominal pain. The clinical picture may be indistinguishable from acute appendicitis. Often the final diagnosis is made at laparotomy, when characteristic findings indicate Crohn disease. Diverticulitis is high on the list of differential diagnoses, especially when the patient has a known history of diverticulosis. Fever, acute onset of abdominal pain, and leucocytosis are consistent with diverticulitis. However, the location of diverticulitis is typically in the left lower quadrant. In the case of redundant sigmoid colon, however, the pain can be located in the right lower quadrant. Diarrhea is often present in diverticulitis. Colon cancer would not present in this fashion, and her abdominal pain with the CT findings does not support viral gastroenteritis.

A family is enjoying a cruise of the lower Caribbean islands. All members of the family went on a shore excursion and visited a local restaurant. They all ate off the "exotic Caribbean" buffet. Four of five members of the family became ill almost simultaneously about 2 hours after eating. All developed nausea with profuse vomiting. All ill family members ate salad greens, rice, and cold macaroni salad. No one else on the cruise ship is currently experiencing these symptoms. Which of the following is the most likely pathogen? A) Norovirus (formerly known as Norwalk virus) gastroenteritis B) Bacillus cereus C) Vibrio vulnificus D) Enterohemorrhagic E. coli

Answer B. Bacillus cereus Explanation The rapid onset suggests a preformed toxin that is seen with Bacillus cereus or Staphylococcus aureus food poisoning. Nausea with profuse vomiting is classic for this, and the incubation period is short—anywhere between 1 and 6 hours for most cases. Rice and other starches left out at room temperature for a period of time are classic for B. cereus toxin to develop. Norovirus would cause a much wider ship-wide outbreak and would be staggered over time as more people came in contact with the virus. Vibrio vulnificus would cause wound infection and primary sepsis, not a nausea/vomiting illness. Enterohemorrhagic E. coli would cause a bloody diarrheal illness with subsequent possible development of renal insufficiency; the incubation period is on average 3-4 days.

Which of the following is the most common cause of acute fulminant liver failure in the United States? A) Hepatitis B virus B) Drug hepatotoxicity C) Hepatitis C virus D) Wilson disease E) Ingestion of amanita species mushrooms

Answer B. Drug hepatotoxicity Explanation If one includes acetaminophen, then drug hepatotoxicity is clearly the most common cause of acute liver failure in the United States. Acetaminophen is the single most important causative agent, although many other drugs may cause this as well. In cases of acetaminophen hepatotoxicity, many are due to suicide attempts. However, there are many others who have this due to accidental toxicity during attempts at pain relief. This can occur if the person over several days is ingesting relatively large doses of acetaminophen. Alcoholics seem to be at risk for toxicity even at lower than usual levels of acetaminophen ingestion. Suicide patients often present promptly, but the accidental toxicity patient may have a delayed presentation, and therefore often has higher mortality. If the patient does present early on, then N-acetyl-cystine may be helpful if administered promptly. Some cases do progress to absolute liver failure, which would require transplantation.

A 40-year-old female patient routinely sees a rheumatologist for scleroderma. She now complains of new-onset dysphagia. Actually, the symptoms have been gradually increasing over the past 4 to 6 months. She denies any prior heartburn or nocturnal regurgitation. The dysphagia is present for meats and breads. It has never been noted for liquids. Past Medical History: Significant only for the scleroderma Review of Systems: Significant for no weight loss or chest pain Physical Examination: Remarkable for the typical findings of scleroderma on the face and hands You recommend which of the following? A) Empiric metoclopramide before meals because of the likelihood that gastroparesis is a factor in reflux. B) EGD and dilate if there is a narrowed area of the esophagus. C) Avoid endoscopy because of increased risk of perforation in scleroderma. D) Empiric use of omeprazole and re-evaluate in 2 months.

Answer B. EGD and dilate if there is a narrowed area of the esophagus. Explanation Scleroderma patients are at risk for reflux, with associated complications such as ulcerative esophagitis, esophageal stricture, and Barrett esophagus. It is very important to establish if any complications exist, for there is benefit to long-term medical treatment. Any patient with these complications would need to be on lifelong proton pump inhibitor therapy. There is no increased risk associated with these procedures in a patient with scleroderma. Remember, of course, that the classic way to approach dysphagia is with the barium swallow test first, although many begin with EGD if there appear to be no contraindications.

You evaluate a 17-year-old boy who has been generally healthy all of his life. He has a history of acne and was prescribed doxycycline. He presents now with a sudden onset of pain with swallowing of both liquids and solids that has been present for 2 days. He is weak and having difficulty sleeping because of the discomfort. He has not suffered heartburn in the past. His physical exam is significant for the moderate distress and bloodshot eyes from lack of sleep. Which of the following is the most appropriate next step? A) Trial of proton pump inhibitor B) Upper GI, x-ray, and HIV assay C) EGD D) Hyoscyamine PO before meals E) Supportive care

Answer E. Supportive care Explanation Doxycycline is one of the most common etiologies of pill esophagitis. It typically presents when patients take doxycycline at bedtime with a small amount of liquid. It can cause severe odynophagia, leading to severe anorexia with volume depletion. This entity can be diagnosed based on clinical history and does not require imaging studies or lab tests. If the patient fails to improve after a few days of supportive care, EGD should be considered. Proton pump inhibitors do not improve pill esophagitis.

A 40-year-old obese woman presents with heartburn. She also has intermittent dysphagia to solids and liquids, so she undergoes an EGD that shows significant (Stage III) erosive esophagitis. She is started on a PPI. She continues on the PPI for 12 weeks with marked improvement of symptoms. She has a trial off the PPI, and symptoms return within several weeks. Restarting the PPI provides relief. She returns to clinic inquiring as to how long she should take this medicine because she has seen stories on the nightly news about fracture risk. She has no family history of osteoporosis or increased risk of fractures. In addition to making sure she takes in adequate calcium and vitamin D, you tell her to continue her PPI for how long? A) 3 weeks B) Forever C) 3 years D) 3 months

Answer B. Forever Explanation Patients with erosive esophagitis have a 70-80% chance of recurrence without PPI treatment. It is reasonable to attempt a trial off PPI, but she failed that. This patient likely needs lifelong PPI treatment. Data on PPI and fracture risk is controversial, but the FDA does recommend that patients be monitored for increased fracture risk. She has no other risk factors listed (she doesn't smoke, is not thin, and has no family history).

A 30-year-old woman presents with abdominal pain that is relieved by defecation. Her stools alternate from hard to loose and watery, and she frequently has to strain with incomplete evacuation. She reports abdominal distention occurring at least 3 times a week. Nocturnal symptoms occur frequently. Which of her symptoms suggests a diagnosis other than irritable bowel syndrome? A) Stools alternate from hard to loose and watery B) Frequent nocturnal symptoms C) Abdominal distention D) Increased incidence of straining with incomplete evacuation E) Abdominal pain relieved by defecation

Answer B. Frequent nocturnal symptoms Explanation All of the other symptoms are common in irritable bowel syndrome. Nocturnal symptoms are very uncommon and should suggest another diagnosis

A 22-year-old man with a negative past medical history presents with the chief complaint of "turning yellow." He noticed that he was becoming yellow in the eyes yesterday. Today, he noted that his skin was also yellow. He has no nausea, vomiting, or other complaints. PAST MEDICAL HISTORY: Negative SOCIAL HISTORY: Works as a cook. Lives with his girlfriend of 3 months. Became sexually active and had multiple sexual partners starting 1 year ago. Has been monogamous for 3 months and 1 day. Smokes marijuana on weekends. Drinks two or three 6-packs of beer on weeknights. FAMILY HISTORY: Mother, 36, is pregnant. Father, 38, health unknown. Sister, 12, with attention deficit disorder. REVIEW OF SYSTEMS: Essentially noncontributory PHYSICAL EXAM: Only pertinent findings: Scleral icterus; liver edge down about 3-4 cm with a span of 12 cm, slightly tender; spleen tip palpable; no spider angiomas LABORATORY: Anti-HAV IgM Positive Anti-HBc IgM Negative Anti-HBc IgG Negative HBsAg Negative How do you interpret the laboratory data? A) He has acute hepatitis A and past infection with hepatitis B. B) He has acute hepatitis A. C) He has acute hepatitis A and chronic hepatitis B. D) He has chronic hepatitis A and acute hepatitis B. E) He has neither hepatitis A nor hepatitis B.

Answer B. He has acute hepatitis A. Explanation His anti HAV IgM is positive, which indicates acute, active disease. All of his serology for hepatitis B is negative, indicating that he has never had hepatitis B infection. There is no such thing as "chronic hepatitis A." On most "screens" for hepatitis, labs do the following tests: Anti-HAV IgM—looks for acute hepatitis A HBsAg—looks for acute infection, as well as chronic carrier states Anti-HBc IgM—looks for acute infection in the "window" Anti-HBc IgG—tells you if a person has been infected with hepatitis B in the past; does not tell you if they are still infectious—this requires the HBsAg test Hepatitis C antibody—tells you if person has been infected with hepatitis C; it does not tell you status of infection (i.e., chronically infected or resolved) Regarding the other options: Acute hepatitis A and past infection with hepatitis B: Anti-HAV IgM positive; all of the hepatitis B tests are negative except hepatitis B Core IgG antibody. Chronic A and Acute B: Impossible; remember that chronic A does not exist. Chronic A and Chronic B: Again ... impossible to have chronic A. Neither hepatitis A nor hepatitis B: This would be correct if all the antibodies studies were negative.

A 65-year-old female presents to the clinic with diarrhea that has been present for the last 7 days. The stool is watery and no blood has been noted. She is not toxic appearing, and her WBC is normal. She does not have fever. Her recent history is important for a skin abscess that was treated with a 10-day course of antibiotics 5 weeks ago. Stool studies reveal normal flora on cultures with no significant blood or fecal leukocytes noted. Evaluation for C. difficile toxin is positive. Which of the following is the most appropriate intervention at this time? A) Recommend over-the-counter loperamide. B) Institute a course of oral metronidazole. C) Prescribe a course of oral clindamycin. D) Admit for intravenous vancomycin.

Answer B. Institute a course of oral metronidazole. Explanation Clostridium difficile is capable of producing a toxin that leads to persistent diarrhea typically, but not always, following a course of antibiotics. Clindamycin, penicillin, and cephalosporins are the most commonly associated antibiotics with C. difficile diarrhea. Motility inhibitors are not recommended in toxin-mediated diarrhea due to the risk of toxic megacolon. Vancomycin and metronidazole are used to treat C. difficile diarrhea though metronidazole is the drug of choice. Oral ( not IV) vancomycin would be reserved for more severe disease with an elevated WBC or toxicity.

A 42-year-old man presents with difficulty swallowing solid food. He says that the symptoms began over 3 years ago. In the beginning, he noted it only with trying to swallow large pieces of steak without chewing properly. Since then, it has gradually worsened to the point that he must chew each piece of food carefully and use lots of water to swallow. He has not had weight loss. He denies fever or night sweats. He has not had any nausea, vomiting, or recent abdominal pain. Social History: He does not smoke and does not drink alcohol. Which of the following is the likely etiology of his dysphagia? A) Squamous cell carcinoma of the esophagus B) Peptic stricture C) Adenocarcinoma of the esophagus D) Schatzki ring E) Achalasia

Answer B. Peptic stricture Explanation This is a very gradual progressing dysphagia. Note that if it had occurred over several months with weight loss, cancer would be the likely culprit. Schatzki rings are very common, but they are non-progressive.

A 21-year-old man is referred for abdominal pain for 5 years. He has had a negative workup from an outside gastroenterologist including CT, retroperitoneal Doppler, EGD and colonoscopy (both with biopsies), and videocapsule endoscopy. What is the next most appropriate step? A) Celiac disease serology B) Questioning about sexual and physical abuse C) Repeat entire workup D) Serum gastrin level

Answer B. Questioning about sexual and physical abuse Explanation This patient likely has severe irritable bowel syndrome (IBS), and questioning about abuse history will aid in identifying the most severe and treatment refractory subjects. He has already had biopsies and did not show evidence of celiac disease. This is not a gastrinoma because he has not had any ulcers documented. It would be useless to repeat the workup again with all negative studies.

You are asked to consult with a 31-year-old man for the evaluation of jaundice. Two days ago, he underwent operative repair of a right medial meniscus tear, and yesterday the surgeon noted the patient became yellow. The patient is asymptomatic and has never had chronic health problems. He takes no medications except Percocet® for postoperative pain, and he has had no known exposures or travel outside the U.S. He is up-to-date with immunizations. He drinks 2-3 beers a week. He does not use any illicit drugs. ROS: He recalls similar eye discoloration after "the flu" 2 years ago. Physical exam is normal except for scleral icterus and jaundice, and surgical dressings on the right knee. Laboratory results: HBsAg: Negative Anti-HBs antibody: Positive Anti-HBc (IgM): Negative HAV ab: Negative AST: 40 U/L ALT: 36 U/L Alk Phos: 110 U/L Bili (T): 3.2 mg/dL Bili (D): 0.4 mg/dL Complete blood count and basic metabolic panel are normal. Urinalysis is negative. What is the most appropriate next step in patient care? A) Liver biopsy B) Reassurance C) Right upper-quadrant ultrasound D) Initiation of pegylated interferon plus ribavirin E) Endoscopic retrograde cholangiopancreatography

Answer B. Reassurance Explanation This patient's clinical presentation is most consistent with Gilbert syndrome, a benign cause of hyperbilirubinemia. Approximately 9% of the population is homozygous for the disorder, diagnosed after some physiologic stressor causes increases in serum bilirubin to levels high enough to warrant laboratory evaluation. The lack of known exposures, travel, drug use, and minimal alcohol use, as well as the normal exam and hepatitis serologies (the anti-HBs antibody was likely from childhood vaccination), make other diagnoses much less likely; thus, further invasive evaluation at this time is not warranted. These patients may simply be followed clinically and informed that febrile illnesses, some medications, dehydration/starvation, and other stressors may increase the bilirubin to noticeable levels (baseline bilirubin in Gilbert's patients is usually < 3.0 mg/dL; it may rise to 6.0 mg/dL).

A 16-year-old male presents with the "flu" and jaundice. The parents seem unconcerned by the jaundice. They relate that he "does this" every time he becomes ill. In fact, his father and uncle do the same thing. You convince them to check their son's bilirubin, which comes back with indirect bilirubin of 4 mg/dL. The remainder of the liver tests are normal, as is a complete blood count. Which of the following would be the next appropriate step? A) Liver ultrasound B) Reassurance that this is a benign condition C) Liver biopsy D) Hepatitis panel

Answer B. Reassurance that this is a benign condition Explanation The history and lab supports a diagnosis of Gilbert syndrome. This is a benign condition in which the patients become jaundiced during stress and fasting, but it causes no pathology. The diagnosis of this inherited disorder is supported by the family history of relatives who follow a similar pattern.

Following an evaluation for unexpected hepatic dysfunction, a patient in your practice is diagnosed with hemochromatosis. Which of the following is the most effective therapy for hemochromatosis? A) Exchange transfusion B) Repetitive phlebotomy C) Intravenous deferoxamine D) Interferon therapy

Answer B. Repetitive phlebotomy Explanation Hemochromatosis is an iron metabolism disease where excess iron is stored intracellularly and leads to significant tissue damage involving liver, adrenal, and testicular tissue. Restriction of ingested iron is an important intervention that is coupled with an induced iron deficiency state. This is best accomplished by repetitive phlebotomy. Deferoxamine is useful in acute iron toxicity, but it is not effective in mobilizing intracellular iron stores. Exchange transfusion removes circulating iron but also replaces it in the form of fresh red blood cells. Interferon therapy is beneficial in treating liver damage resulting from viral hepatitis infections but has no role in hemochromatosis-induced liver damage.

A 45-year-old male is accompanied to the clinic by his wife. He complains of difficulty swallowing for several years, but his wife has become increasingly concerned about his choking, particularly when eating with guests in a restaurant. It is nonpainful and intermittent but occurs most frequently when he is eating meat. Other solids are usually, but not always, tolerated. He has never had difficulty with liquids. He denies any reflux or heartburn symptoms. He has had no weight loss. Which of the following is the most likely diagnosis? A) Barrett esophagus B) Schatzki ring C)Diffuse esophageal spasm D) Celiac sprue E) Achalasia

Answer B. Schatzki ring Explanation The key points in this scenario are the nonpainful, nonprogressive dysphagia to solids but not liquids. Diffuse esophageal spasm is typically very painful (nutcracker esophagus) and involves both liquids and solids. Likewise, achalasia usually affects both liquids and solids. Barrett esophagus is related to damage to the distal esophagus and is commonly—but not always—related to heartburn symptoms. It is pre-malignant and can lead to neoplastic growths that increase over time. The nonprogressive nature of this patient's complaint makes this option less likely. A Schatzki ring is a lesion of the esophagus that does not usually progress and is not frequently associated with painful dysphagia. The size of the ring will determine the size of swallowed bolus that can pass. The frequency of a meat bolus resulting in symptoms has led this condition to also be referred to as "steak house dysphagia." Celiac sprue does not have esophageal symptoms as its primary symptom.

A 19-year-old female who has recently been diagnosed with celiac disease presents to your clinic to establish care. During the visit, she seeks your advice concerning the anticipated course of her celiac disease. Which of the following is true regarding her condition? A) Rye or barley products should replace wheat products. B) She is at increased risk for Addison disease and lymphomas. C) Dapsone therapy will reduce her gastrointestinal complaints. D) Pancrease supplements will improve the malabsorption effects.

Answer B. She is at increased risk for Addison disease and lymphomas. Explanation Celiac disease, nontropical sprue, or gluten sensitivity is an inflammatory response to dietary gluten that damages the intestinal mucosa. Gluten is found in several grains, including wheat, rye, and barley, as well as many commercial products that add gluten as a food stabilizer. Malabsorption of nutrients, vitamins, and minerals is a common complication, but because the defect stems from mucosal damage, pancreatic enzyme replacement has little benefit. Celiac disease is linked to dermatitis herpetiformis, which the skin lesions respond well to dapsone therapy. Dapsone, however, is ineffective in controlling intestinal symptoms. Celiac disease is also associated with an increased risk for non-Hodgkin lymphoma, thyroid and esophageal cancer, Addison disease, osteoporosis, neuropathies, and Type 1 diabetes mellitus.

A 68-year-old woman presents with complaint of difficulty swallowing. She notes that about 4 months ago she had trouble swallowing big pieces of food; this has progressed to the point that she has difficulty swallowing thick liquids. She has lost about 10 pounds and attributes this to a lack of desire to eat. She denies heartburn or other gastrointestinal symptoms. Past Medical History: Significant for 2 packs of cigarettes per day for 40 years Physical Exam: She is alert and oriented to person, place, and time. She is pale in appearance. She has indeed lost 12 pounds since you last saw her. Otherwise, her examination is normal. Which of the following is the most probable etiology of her weight loss and dysphagia? A) Schatzki ring B) Squamous cell carcinoma of the esophagus C) Adenocarcinoma of the esophagus D) Peptic stricture E) Achalasia

Answer B. Squamous cell carcinoma of the esophagus Explanation Dysphagia for solids that is rapidly progressive (over a few months) and accompanied by anorexia and weight loss suggests esophageal carcinoma. Squamous cell carcinoma of the esophagus is associated with smoking, achalasia, and a history of lye ingestion. Adenocarcinoma would be suspected if she had a history of Barrett esophagus or GERD. Schatzki ring is a common cause of non-progressive dysphasia in younger people. Peptic stricture would cause a very gradual worsening of dysphasia but only to solids and not to liquid.

A 28-year-old internal medicine resident has been healthy all of his life. He recently developed acne, which was attributed to the copious ingestion of Hershey bars on call nights. He prescribed for himself doxycycline. He presents now with a sudden onset of pain with swallowing anything, either liquid or solid. This has been present for 2 days. He is weak and is having difficulty sleeping because of the discomfort. He has rarely suffered heartburn in the past. His physical exam is significant for the moderate distress and some blood-shot eyes from lack of sleep. You recommend which of the following? A) Hyoscyamine PO before meals B) Supportive care C) EGD D) Trial of proton pump inhibitor E) Upper GI, x-ray, and HIV assay

Answer B. Supportive care Explanation Doxycycline is one of the medicines most commonly mentioned as a cause of pill esophagitis. It presents typically when patients take it at bedtime with a small amount of liquid to rinse down. It will cause severe odynophagia almost to the point where the patient can have little or no intake. Neither the EGD nor the upper GI is going to add anything to the patient care at this time, other than provide interesting pictures. Likewise, since reflux has no role in this scenario, use of the PPI would be of little benefit.

A 27-year-old male presents to the emergency department with complaints of rectal pain that has been progressing over the last 24 hours. He states he has been feverish with some nausea and has pain that is much worse with defecation. On physical examination, his temperature is 101.8° F (38.8° C). Bowel sounds are present and the abdomen is soft. On examination of the rectum, a 1.3-cm pustule is noted at the anal verge with surrounding erythema and tenderness. Digital rectal exam is not possible due to the exquisite tenderness. Based on this information, which of the following is the most important intervention for this patient? A) Diagnostic sigmoidoscopy B) Surgical exploration and debridement C) Incision and drainage of the pustule D) Admission for IV antibiotics

Answer B. Surgical exploration and debridement Explanation Infections in the perianal/perirectal area have a similar presentation of rectal pain but differ significantly in treatment approach. Perianal abscesses are more superficial and lack systemic findings. With adequate analgesia, a perianal lesion can be treated with incision and drainage. On the other hand, a perirectal abscess is more extensive and typically demonstrates fever, leukocytosis, and a marked tendency to fistula formation. Diagnosis can be made by ultrasound, but CT scan is much more sensitive. Sigmoidoscopy would be impractical with the levels of pain experienced with the abscess and is not an effective method to evaluate a perirectal abscess. Antibiotic usage in these conditions is subject to debate, but they are clearly inadequate to resolve the complaint. Patients with immune compromise are at increased risk for a perirectal abscess but most patients have no specific risk factor. The definitive intervention for a perirectal abscess is surgical exploration and debridement of all necrotic tissue.

A 24-year-old male, currently in his 6th year in college, describes intermittent diarrhea for 3 years. The loose stools can alternate with constipation. He never has nocturnal stools. He suffers with mild abdominal cramps. PMH is negative. Review of Systems: Negative for weight loss, fever, joint pain, skin rash Social History: Beer on weekends, 2 glasses milk each day. He constantly chews diet gum. No travel. Which of the following is the best plan of action at this time? A) 72-hour fecal fat collection B) Trial of lactose-free, sorbitol-free diet C) Colonoscopy D) CT of abdomen and pelvis

Answer B. Trial of lactose-free, sorbitol-free diet Explanation This is probably irritable bowel syndrome. There are no red flags to prompt an aggressive workup. Remember that lactose and sorbitol can always be factors. On the Boards, look out for young people who are healthy, on who they try and get you to do invasive tests when they don't need them. The one main exception in gastroenterology, however, is to be on the lookout for a young patient with undiagnosed Crohn's or ulcerative colitis who needs an invasive test to make the diagnosis (however, symptoms and signs will be present for you to go to that procedure).

A 17-year-old female presents for evaluation of chronic abdominal pain. She describes the pain as a chronic, colicky pain in the right lower quadrant. She has had no weight gain for a year and has delayed puberty. Physical Examination: Thin, pale adolescent. She has no secondary sexual development. Her abdomen is slightly distended. There are good bowel sounds but mild tenderness in the right lower quadrant and a palpable loop of bowel. The stool from the rectal tests is heme-positive. The stool has red and white blood cells. The stool cultures, O&P, and C. difficile toxin are all negative. Which of the following studies is most likely to confirm your diagnosis? A) Abdominal CT scan B) Upper and lower endoscopy with mucosa biopsy C) Barium enema D) Upper gastrointestinal series with small bowel follow-through

Answer B. Upper and lower endoscopy with mucosa biopsy Explanation The patient in the case clearly has a history suggesting Crohn disease: colicky abdominal pain, growth failure, and delay of sexual maturation. Although the findings on all of the radiological studies can strongly suggest Crohn disease, the definitive diagnosis is made by finding granulomas in a mucosa biopsy.

A 55-year-old Caucasian woman presents with history of abdominal complaints off and on "for years." She presents with acute onset of severe pain in her left abdomen. She has fever to 101° F at home and appears ill. She has had some associated diarrhea with the pain. PAST MEDICAL HISTORY: History of GI disease SOCIAL HISTORY: Housewife; volunteers at her church quite often Lives at home with her husband FAMILY HISTORY: Mother died at age 78; renal failure Father died at age 25; industrial accident REVIEW OF SYSTEMS: No headache No sore throat No runny nose; congestion Occasional dry cough No tachycardia No chest pain No constipation Diarrhea frequently, usually with emotional upset No burning on urination PHYSICAL EXAMINATION: BP 130/88, P 100, RR 18, Temp 101.5° F Ill-appearing woman in mild distress HEENT: PERRLA, EOMI TMs clear Throat clear Neck: Supple; no masses Heart: RRR without murmurs, rubs, or gallops Lungs: Clear to auscultation Abdomen: Hyperactive bowel sounds; tender and palpable mass in left lower quadrant; rebound tenderness noted and involuntary abdominal rigidity noted Rectal: Heme-positive Extremities: No rashes, cyanosis, or edema LABORATORY: Pending Endoscopy in 2000 Based on her physical findings, which of the following is the most appropriate next step? A) Emergent barium enema. B) Bowel rest only is adequate at this point. C) Abdominal CT scan. D) Bleeding scan. E) Emergent colonoscopy.

Answer C. Abdominal CT scan. Explanation She has known history of diverticulitis, and now with the findings of rebound tenderness and involuntary abdominal rigidity, there is the possibility of an abscess or perforation of a diverticulum. Emergent CT scan (or ultrasound) should be done to evaluate for this possibility. Colonoscopy and barium enema should be avoided during the active stage. If an abscess is found, drainage is necessary either with radiologic guidance or surgical intervention. Bowel rest is indicated, but you must rule out the possibility of something more severe such as abscess or perforation. A bleeding scan is not indicated since she has no evidence of a severe bleed.

A 65-year-old woman presents to the office with the complaint of fatigue for 2 months. She has no specific pain or discomfort. She has noted no change in her bowel habits. Past Medical History: Significant for hypertension and hyperlipidemia. She avoids aspirin and NSAIDs. Family History: Non-contributory Physical Exam: This is unremarkable except for the presence of heme-positive brown stool. Laboratory: Hemoglobin is 7.8 with an MCV of 65. You refer to the local gastroenterologist, who does an EGD. The EGD is normal. A colonoscopy is also performed. Unfortunately, she had a very tortuous colon, and the exam was only to the hepatic flexure. The patient was then sent back to you for consideration of additional studies. Given what you know now, what should you do next for this patient? A) No further studies; start the patient on iron and have her return to clinic in 2 months for a repeat hemoglobin check. B) Send for colonoscopy to a different gastroenterologist. C) Air contrast barium enema. D) Upper GI small bowel series.

Answer C. Air contrast barium enema. Explanation Our major concern with this woman is colon cancer, and the right colon has not been excluded by the incomplete colonoscopy. This area must be evaluated before the colon can be pronounced clear of pathology. During the initial evaluation, colonoscopy was the best test, and in experienced hands one should be able to view the entire colon in approximately 98% of exams. For the remaining small percentage of incomplete exams, it is important as a first priority to evaluate the rest of the colon.

A 50-year-old woman of Irish descent is referred to your office after another internist noted microcytic anemia on a routine test. She has no specific complaints. She describes 2 soft and slightly loose bowel movements every day, but this has been her normal pattern for many years. It has not progressed in any way. She denies any abdominal pain, although admits to some nonspecific bloating, again present for many years. PAST MEDICAL HISTORY: She is on thyroid replacement for hypothyroidism; she is status-post hysterectomy and is on replacement estrogen as well FAMILY HISTORY: Negative for any gastrointestinal disease SOCIAL HISTORY: Negative REVIEW OF SYSTEMS: She denies fever, chills, weight loss, chest pain, or dyspnea PHYSICAL EXAMINATION: Unremarkable except for slight paleness to the skin HEENT: Sclera anicteric Chest and Cardiac exam: Normal Abdomen: Soft and nontender without organomegaly; stool brown and heme-negative LABS: Hgb 9.0, MCV 70, serum iron 8, TIBC 400 You order stools for occult blood and these are negative x 3. Which of the following should you recommend? A) Order upper GI small bowel series. B) IgG antigliadin and antiendomysial antibodies; if positive, treat with steroids and gluten-free diet. C) Colonoscopy and endoscopy. If the latter is grossly normal, obtain oriented biopsies of the duodenum. D) Flat plate x-ray of the abdomen and a trial of pancreatic enzymes if calcifications are found.

Answer C. Colonoscopy and endoscopy. If the latter is grossly normal, obtain oriented biopsies of the duodenum. Explanation The patient has celiac disease. This has led to the bloating and loose stools, which represent malabsorption, as well as her iron deficiency anemia. The iron deficiency anemia may not be due to occult bleeding, but rather interference with the absorption of iron, which takes place mainly in the duodenum. The duodenum is the region in the small bowel most affected by celiac disease, and it is not uncommon for these patients to be iron-deficient. However, performing a colonoscopy and endoscopy is correct because in her age group, a full endoscopic evaluation needs to be done on anybody with an iron deficiency anemia, even in the absence of heme-positive stools. Clearly, colonoscopy needs to be done to rule out colon cancer. On upper exam, there are some characteristic findings that can indicate celiac disease. The duodenal folds may be atrophic or have a scalloped appearance. However, even in the absence of endoscopic findings, one should do the oriented biopsies of the duodenum if one suspects celiac disease. The upper GI series would not be accurate enough to show any specific changes for celiac disease. The antigliadin and antiendomysial antibodies can be helpful in diagnosing patients with celiac disease, but today the best test is either IgA (not IgG) antiendomysial antibody or tissue transglutaminase (tTG) antibody. The antigliadin antibody is nonspecific. Also the gold standard of diagnosis stills relies on duodenal biopsies, and one should have that before making the diagnosis and initiating therapy. Additionally, steroids are not indicated at this time. In this case, one would simply treat with a gluten-free diet.

What is the observed mortality benefit from screening colonoscopy due to? A) Removal of hyperplastic polyps B) Detection of bacterial overgrowth C) Detection of incident cancers D) Removal of adenomas

Answer C. Detection of incident cancers Explanation Despite many attempts to demonstrate a survival benefit from removal of adenomas, colonoscopy has only ever been shown to improve survival because of detection of incident neoplasms. Hyperplastic polyps and bacterial overgrowth do not have inherent increased cancer risk.

A 40-year-old patient has had ulcerative colitis since age 20. In the past 5 years, he has been completely asymptomatic and in remission since the initiation of therapy with azathioprine. His bowel movements are normal, and he never notices rectal bleeding. He further denies any abdominal pain. PAST MEDICAL HISTORY: Significant only for ulcerative colitis; his medications include sulfasalazine 2 g/day and azathioprine 100 mg/day FAMILY HISTORY: Positive for 1 cousin with Crohn disease REVIEW OF SYSTEMS: Denies any jaundice, itching, or abdominal pain LABS: Alk phos 410, AST 50, ALT 68, bili 2.2, d. bili 1.8, Hgb 12, WBC 5,000 Ultrasound reveals a normal-sized liver and no enlargement of the bile ducts. Which of the following studies should you recommend as the next best step? A) Abdominal CT scan B) Laparoscopic cholecystectomy C) ERCP D) Liver biopsy

Answer C. ERCP Explanation This patient with ulcerative colitis has likely developed primary sclerosing cholangitis. Of all patients with PSC, 70% will have a history of ulcerative colitis. As in the case of this patient, there is a strong male predominance. Many of these patients are asymptomatic at the onset. Some will give symptoms of cholangitis, such as fever and chills, and some may present with fatigue and malaise. At the time of ERCP, there may be multiple strictures of both the intra and extrahepatic bile ducts. There often is not enough dilatation for this to be noted on studies such as the ultrasound. Dominant strictures of the bile duct can be dilated at the time of ERCP. There is no additional value to a liver biopsy at this time. An abdominal CT scan would likely not add any more than the ultrasound already did. And finally, there is no indication for laparoscopic cholecystectomy either.

A 66-year-old male is seen in the clinic after referral from his dentist due to the recent development of halitosis. His dental exam was unremarkable. The patient states that he has experienced increasing difficulty swallowing that worsens as he progresses through a meal. He also relates that he regurgitates food that appears undigested hours after a meal. He has lost 5 pounds over the preceding 6 months. A barium swallow reveals a 6-cm posterior pouch in the upper esophagus. The mucosa appears smooth, and the remainder of the exam is reported as normal. Which of the following statements is true regarding this patient? A) Direct endoscopic examination is required. B) Institution of a motility agent will help relieve symptoms. C) Effectiveness of medications may be reduced in this condition. D) Esophageal manometry will aid in treatment decisions.

Answer C. Effectiveness of medications may be reduced in this condition. Explanation Zenker diverticula are uncommon outpouchings of the posterior esophageal wall primarily in patients older than 50 years of age. As the pouch enlarges, it tends to accumulate swallowed food and may enlarge enough to obstruct further ingestions. The accumulated material leads to halitosis and delayed regurgitation. The associated dysphagia can also result in decreased nutrition and weight loss. There is a low association of Zenker diverticula with squamous cell neoplasms. Irregularities on barium studies should prompt direct endoscopic visualization, but the risk of perforation during endoscopy limits its indications. The dysphagia is not related to motility problems and motility stimulants would have no benefit. The differential for dysphagia with regurgitation of undigested food includes achalasia, but this would be unlikely with this barium study result. Oral medications are occasionally trapped in a Zenker diverticulum preventing absorption and often complete loss of the dosage through regurgitation. Both of these effects can significantly reduce the response to a medication regimen for any medical condition.

A 60-year-old Native American woman with a history of hypertension and hyperlipidemia presents with right upper quadrant pain and fever. She has been ill for several days and reports that she has been vomiting on occasion during the last day or so. She denies recent travel. PAST MEDICAL HISTORY: Hypertension for 20 years, currently treated with fosinopril 20 mg q day SOCIAL HISTORY: Smokes 1/2 pack a day for 45 years Alcohol: Occasional glass of wine on weekends FAMILY HISTORY: Mother died from acute MI at age 75 Father died from "old age" at 99 Sister with gallstones and HTN Brother with HTN REVIEW OF SYSTEMS: Decreased appetite, recent fatty food intolerance; fever for 2 days, chills PHYSICAL EXAMINATION: General: Moderately ill-appearing woman VS: Temp 102.2° F, BP 110/70, Pulse 105, RR 25 HEENT: Scleral icterus, PERRLA, EOMI Throat: clear Heart: RRR with no murmurs, rubs, or gallops; tachycardic Lungs: CTA Abdomen: Diminished bowel sounds, right upper quadrant tenderness to palpation; she has rebound tenderness also GU: Normal female genitalia, no tenderness on bimanual palpation Extremities: No cyanosis, clubbing, or edema LABORATORY: White Blood Count (WBC): 18,500 with 70% neutrophils, 15% band forms, 10% monocytes Hematocrit: 37.2% Platelet count: 522,000/mL Serum chemistries: Sodium 140 mg/dL, chloride 110 mg/dL, potassium 4.2 mg/dL Total bilirubin 6 mg/dL with a direct bilirubin of 4 mg/dL Serum aminotransferases: AST 90 IU/L, ALT 75 IU/L; alkaline phosphatase 300 IU/L Computed tomography of the abdomen shows a dilated common bile duct and no other abnormalities. The next appropriate diagnostic study is which of the following? A) Liver biopsy B) Magnetic resonance imaging of the biliary system C) Endoscopic retrograde cholangiopancreatography (ERCP) with laparoscopic cholecystectomy D) Ultrasonography E) Exploratory laparotomy

Answer C. Endoscopic retrograde cholangiopancreatography (ERCP) with laparoscopic cholecystectomy Explanation Complications of gallstones include acute cholecystitis, biliary colic, gallstone ileus, fistula formation, porcelain gallbladder caused by calcium and salt deposition in the wall, and stones in the common bile duct, which occur in up to 10-15% of patients. Occult duct stones remaining after cholecystectomies can occur in up to 1-5% of patients. Rarely, primary stones will occur in the ducts in the setting of increased pigment or congenital abnormalities. This patient has symptoms of acute cholangitis (Charcot triad: biliary colic, jaundice, and fever with chills). Cholangitis is inflammation of the bile ducts usually caused by bacteria and most often occurs when there are gallstones partially obstructing the bile tract. Many patients with this condition respond rapidly with appropriate supportive measures, including antibiotics. The concern is when a complete obstruction of the ductal system occurs, which can lead to severe illness with resulting SIRS (systemic inflammatory response syndrome) or septic shock. The most appropriate diagnostic study for choledocholithiasis (gallstones) is cholangiography, which usually is accomplished by ERCP with combined laparoscopic cholecystectomy. This reduces the risk of complicated biliary tract disease with the need for choledocholithotomy and T-tube drainage. Ultrasound and MRCP would not be appropriate for an urgent scenario. Liver Bx would not be appropriate. One other pearl from this case: A palpable gallbladder (Courvoisier sign) would suggest carcinoma of the pancreas.

A 30-year-old man with a 1 year history of gastroesophageal reflux disease (GERD) returns for follow-up. He has a lengthy history with you as follows: You first saw him a little over 9 months ago and recommended that he sleep with the head of his bed elevated by 6 to 9 inches, lose 10-15 pounds of weight, eat small meals, and eat dinner 3 hours before bedtime. You additionally strongly recommended that he stop smoking. At that time, you started him on ranitidine 150 mg twice daily. He returned 1 month later saying that he had done everything that you recommended, but his GERD was still quite severe. You started him on omeprazole 20 mg daily and discontinued the ranitidine. He returned a month after that and said his symptoms were markedly improved and that he could sleep flat without discomfort. He continued to lose weight and also quit smoking. You recommended he complete a 3-month course of therapy with omeprazole. He returned 4 months later (now, off of omeprazole) and told you that his symptoms had all returned and actually seemed worse. You restarted his omeprazole for another 3-month course. His symptoms resolved and he was doing well. He now returns again today, 2 weeks off therapy, reporting that his symptoms have returned again. PAST MEDICAL HISTORY: Nothing significant except for motor vehicle accident when he was 18 SOCIAL HISTORY: Quit smoking as you recommended about 9 months ago Alcohol: Glass of wine on the weekend Employment: Works at a donut shop Started exercising 6 months ago, now runs 3 miles every other day FAMILY HISTORY: Mother 65, healthy Father 66, known coronary artery disease, diabetes Brother 32, GERD ROS: Non-contributory PHYSICAL EXAMINATION: Unremarkable, very healthy 30-year-old man LABORATORY: EGD done 11 months ago showed grade 3 esophagitis (circumferential erosions and exudative lesions) He tells you today that he is tired of taking these medications and realizes that he may need to take them chronically. He asks you what would be his best option for treatment of his GERD, based on his age and current health status. Which of the following should be your response? A) Because he is young, the best therapy is over-the-counter medications such as calcium-containing antacids, because they are much safer than the proton pump inhibitors. B) Because he is young, it may be best to begin metoclopramide for long-term therapy. C) He has severe disease that will likely require long-term medical therapy, so the best option is chronic proton pump inhibitor therapy. D) Even though he has frequent recurrences, it is likely that as he gets older these episodes will become less frequent; therefore, medical management with omeprazole is the most prudent choice to see if his GERD will resolve over time. E) Omeprazole cannot be used for long-term maintenance therapy.

Answer C. He has severe disease that will likely require long-term medical therapy, so the best option is chronic proton pump inhibitor therapy. Explanation The 2008 GERD guidelines recommend medical therapy over surgical intervention because of the inherent risks of surgery and the potential for increased side effects such as bloating and GI discomfort after surgery. It is unlikely that his GERD, severe as it is, will improve or "resolve over time." Because of its side effects (tachyphylaxis, lethargy, extrapyramidal effects, etc.), metoclopramide cannot be used long-term. As he has already shown, non-proton pump inhibitors are not likely to be effective. Omeprazole or another proton-pump inhibitor could be used for long-term therapy and is an option.

A 35-year-old male IV drug abuser presents with new onset of jaundice. His immunization history is unknown. Laboratory returns with the following results: Anti-HAV IgM: Negative Anti-HAV IgG: Positive HBcAb-IgM: Negative HBcAb-IgG: Positive HBsAg: Positive Anti-HDV IgM: Positive Anti-HDV IgG: Negative Which of the following is the most likely diagnosis? A) Acute hepatitis A infection. B) Acute hepatitis A, acute hepatitis B, and acute hepatitis D infection. C) Hepatitis D infection. D) He has never had a hepatitis A infection. E) He has recovered from a recent hepatitis D infection.

Answer C. Hepatitis D infection. Explanation He may have coinfection with hepatitis B and D or a superinfection with just hepatitis D. Remember that for hepatitis D to occur, the patient must have circulating hepatitis B surface antigen (HBsAg). He had hepatitis A in the past and is now immune. He is infectious for hepatitis B because his surface antigen is positive, but we cannot say if he is a chronic carrier based on the information presented; this requires follow-up after six months to assess whether the patient has developed antibodies against the surface antigen and effectively "cleared" the virus. His anti-HDV IgG is negative and indicates he has not yet recovered from his hepatitis D infection.

You are moonlighting in the ER when a 30-year-old presents with melena, hematemesis (bright red blood), and orthostasis. She does not drink; she has no stigmata of chronic liver disease; she does not have any risk factors. She has been taking ibuprofen 800 mg tid for a sore ankle. Her initial blood pressure is 60/40; pulse is 113 beats per minute; and her initial hematocrit is 14. Which of the following is the next most appropriate step in management? A) IV normal saline bolus, type and crossmatch, emergent upper endoscopy B) IV D5W and emergent upper endoscopy C) IV normal saline bolus, type and crossmatch D) Call for GI consult and emergent upper endoscopy

Answer C. IV normal saline bolus, type and crossmatch Explanation This patient needs to be stabilized before endoscopy. Her biggest problem is that she is hypovolemic, so she needs IV normal saline and blood quickly. She has had a significant bleed, but taking her to the endoscopy suite in an unstable condition would increase her morbidity and mortality significantly. Generally, always stabilize the patient before performing any type of upper or lower endoscopy. D5W is not an appropriate fluid in someone with volume loss. She needs a volume expander like normal saline. Doing the endoscopy without giving fluids would be a disaster—she would not tolerate the procedure for very long at all.

A 45-year-old man presents with a wasting type illness for the past 6 months. He describes arthralgias and frank arthritis of the larger joints that seem to come and go. He has fever associated with these episodes up to 102° F on occasion. He has had marked episodes of diarrhea over the past 6 months and has lost 20 pounds. Additionally, he describes "swollen glands" for the past few months. PAST MEDICAL HISTORY: Nothing at all; a very healthy man; athletic SOCIAL HISTORY: Lives in Philadelphia, PA Works as a guard at the Liberty Bell Married with 3 children, ages 14, 12, 10 FAMILY HISTORY: Mother 70 with hypertension Father 75 with diabetes, hypertension REVIEW OF SYSTEMS: Difficulty seeing at night—he noticed this in the past month Swelling of the joints noted on occasion Noted increased bleeding of his gums PHYSICAL EXAMINATION: BP 120/60, P64, RR 18, Temp 99.0° F HEENT: PERRLA, EOMI, sclera non-icteric Teeth: Gingivitis present Mouth: Glossitis present Neck: Supple; no bruits Heart: RRR without murmurs, rubs, or gallops Lungs: Clear to auscultation Abdomen: Hyperactive bowel sounds; distended abdomen; no hepatosplenomegaly Extremities: No cyanosis, clubbing, or edema; joint swelling noted of the right knee; no frank deforming arthritis noted GU/Rectal: Mild heme-positive stool; no melena Skin: Diffuse hyperpigmentation, particularly around the orbital and malar face areas LABORATORY: Significant tests for you to consider: Sudan stain of stool shows malabsorption Serum carotene level 5 mg/dL (normal 40-180 mg/dL) Serum albumin 2.4 mg/dL Prothrombin time prolonged at 16 secs Further workup is done and a biopsy is taken of the small intestine that shows the lamina propria contains macrophages containing periodic acid-Schiff (PAS) material. Based on these findings, which of the following is your A) AIDS-related complex B) Mycobacterium avium intracellulare infection C) Infection with Tropheryma whipplei D) Abdominal angina E) Celiac sprue

Answer C. Infection with Tropheryma whipplei Explanation This patient has Whipple disease. Approximately 50% of the patients with Whipple's have generalized hyperpigmentation in association with diarrhea, weight loss, arthritis, and lymphadenopathy. Posterior uveitis is also seen. His night blindness is due to vitamin A deficiency from malabsorption—hence, the low carotene levels. The clincher was the biopsy of the small intestine and the findings in the lamina propria. Also available from certain labs is a PCR for the organism, which could be used on the biopsy material as well.

A 56-year-old male is diagnosed with pancreatic cancer located in the head of the pancreas and impinging on the common duct. Following consultation with oncology and surgery, a Whipple procedure (resection of the head of the pancreas with reanastomoses of the pancreatic and biliary tract) is performed. Following the procedure, which of the following feeding methods offers the best advantage? A) Peripheral nutrition through a PICC line B) Percutaneous gastric (PEG) tube feedings C) Jejunal tube (J-tube) feedings D) Nasogastric (NG) tube gavage feedings

Answer C. Jejunal tube (J-tube) feedings Explanation Assisted nutrition is a vital adjunct in the case of many patients with gastrointestinal pathology. Whenever the enteral path is available, it is the preferred route for nutrition. In this case the lower GI tract is unaffected, making parenteral nutrition unnecessary. NG tube and PEG tube feedings deposit nutritional material in the stomach, which is proximal to the resection region and may delay healing. A J-tube placed during surgery allows adequate nutrition utilizing the functional bowel and permits rest to the healing pancreatic and biliary tree.

A 50-year-old African-American woman presents to GI clinic because of gas, bloating, and diarrhea anytime she drinks milk or eats cheese or ice cream. Symptoms are slightly better when taking an over-the-counter lactase enzyme preparation. Her mother has the same problem. She had a normal colonoscopy last year for colorectal cancer screening. The endoscopist did random colon biopsies that were normal. The next appropriate step is which of the following? A) Celiac antibodies B) Repeat colonoscopy with random biopsies C) Lactose avoidance D) Empiric treatment with metronidazole 250 mg PO tid x 10 d E) Small bowel x-ray to rule out Crohn disease

Answer C. Lactose avoidance Explanation This woman has almost 100% chance of being lactose intolerant (lactase deficient). Lactose avoidance is the best choice of those listed. African-Americans have a much higher prevalence of lactose intolerance than do Caucasians of northern European descent. The first step is to stop lactose ingestion and see if the symptoms improve. If not, then celiac disease would be an option to consider. There is no reason to suspect Crohn disease or to treat her for C. difficile (using the metronidazole). Finally, she does not need colonoscopy with biopsies at this point. Her previous colonoscopy was normal.

A 34-year-old woman presents with a 3-year history of rectal bleeding, abdominal pain, and weight loss. Colonoscopy with biopsy is performed; histology shows acute and chronic mucosal inflammation, crypt distortion with forked glands, and crypt atrophy. You begin sulfasalazine 4 gm daily. Three weeks later, she feels much better, and bleeding has stopped. However, on routine follow-up labs, the following is noted: Laboratory results: WBC: 11,500 3,500-10,000 cells/mm3 WBC differential: Normal ALT: 70 0-35 U/L AST: 65 0-35 U/L Alk Phos: 428 36-92 U/L Total bilirubin: 1.4 0.3-1.2 mg/dL Albumin: 3.1 3.5-5.5 g/dL Total protein: 6.0 6.0-7.8 g/dL BMP: Normal PT: 11.0 11-13 seconds PTT: 20.8 25-25 seconds Urinalysis: Normal RUQ ultrasound is performed; no stones or CBD dilatation are seen. Which of the following findings is most likely to be present? A) Elevated antimitochondrial antibodies and ANA; bridging fibrosis on liver biopsy B) Multiple areas of increased uptake on three-phase bone scan C) Multifocal stricturing and dilation of intrahepatic bile ducts on cholangiography D) HBsAg positive, anti-HBe positive, eventual resolution of liver enzymes E) A large, heterogeneous hepatic mass on contrast CT of the liver

Answer C. Multifocal stricturing and dilation of intrahepatic bile ducts on cholangiography Explanation These findings are diagnostic for primary sclerosing cholangitis (PSC), and 90% of patients with PSC have ulcerative colitis (alternately, 5% of UC patients have PSC). These patients are usually asymptomatic and present with elevated liver biochemical tests (mainly alkaline phosphatase). AMA (antimitochondrial antibody) is positive in primary biliary cirrhosis and usually negative in PSC. Acute viral hepatitis often shows a less obstructed picture: higher liver enzymes and less alkaline phosphatase elevation. This patient has no findings suggestive of bone-associated alkaline phosphatase elevations. Hepatocellular carcinoma may cause elevations of alkaline phosphatase; however, this patient has no risk factors for HCC or history of cirrhosis.

A 32-year-old male patient is new to town and seeks your advice as his new physician. He was diagnosed as having familial polyposis coli at age 16 and had a total colectomy with ileorectostomy at age 19. He has done well since that time. He has 3 bowel movements a day. Family History: Remarkable in that his father and two brothers have polyposis and have all had colectomies. His grandmother died of colon cancer at the age of 40. He has a flexible sigmoidoscopy and many small adenomatous polyps are completely removed. A recent EGD for occasional reflux symptoms was performed. There were multiple large polyps in the body and fundus of the stomach. The duodenum had several tiny, white plaques. One of these plaque-like lesions was biopsied, revealing an adenoma. No dysplasia was found. Because a duodenal adenoma was found, you recommend which of the following? A) Surgery to remove the stomach and duodenum. B) Video capsule to look at the rest of the small bowel. C) Nothing for now, but repeat EGD in 1-2 years. D) Repeat EGD to remove as many of the gastric polyps as possible with a snare technique.

Answer C. Nothing for now, but repeat EGD in 1-2 years. Explanation As predicted, this patient has some small adenomas in the duodenum. He needs close observation but nothing drastic unless the biopsies show high-grade dysplasia or cancer. At endoscopy, close inspection of the ampullary region is important. The gastric polyps noted are benign fundic gland polyps and are not an indication for intervention. One question of interest is whether this patient should be on a COX-2 agent. There is some evidence that they may help in retarding the development of rectal polyps, but there is no information on their role in duodenal polyp formation. Remember: Only well-established standards of care are on the Boards.

A 75-year-old patient presents to the hospital with his second cerebrovascular accident. Although he was functional after his first episode, after 3 days in the hospital, he is still unable to speak, and attempts at swallowing liquids have led to coughing. Which of the following is true? A) Most patients with a PEG placement have severe reflux afterwards; therefore, one should consider a surgical jejunostomy instead. B) Antibiotics are not required before a PEG since this is a sterile procedure. C) Percutaneous endoscopic gastrostomy (PEG) is appropriate intervention to allow hydration and nutrition. D) Upper endoscopy should be done to evaluate the cause of dysphagia prior to any decision on long-term management.

Answer C. Percutaneous endoscopic gastrostomy (PEG) is appropriate intervention to allow hydration and nutrition. Explanation This patient has oropharyngeal dysphagia, of which the most common cause is a CVA. There are other neurological causes as well; e.g., ALS. The best study for this is the modified barium swallow. Endoscopy is not helpful at all in determining the reason for this type of dysphagia. PEG is considered a surgical procedure, and antibiotics do reduce the risk of infection afterwards. Other complications would include bowel perforation, bleeding, and local cellulitis. Many patients will still aspirate after the procedure, although this is more commonly due to aspiration of saliva rather than reflux and aspiration of gastric contents.

A 35-year-old man with ulcerative colitis (UC) presents for follow-up. He has been doing well, except recently he had an exacerbation with severe bleeding and anemia. He has a history of pancolitis, and on his recent colonoscopy, the biopsy showed high-grade dysplasias in flat mucosa. He did not have a mass lesion to biopsy. Previous colonoscopies have shown mild dysplasia with inflammation. Since his recent exacerbation, he has been doing well without any further episodes of bleeding. Of significance is that his last exacerbation prior to this was over 10 years ago. PAST MEDICAL HISTORY: Besides UC, stable SOCIAL HISTORY: Smokes 1/2 ppd x 10 years Abstains from alcohol Married for 10 years Works as a carpet layer FAMILY HISTORY: Mother, healthy, age 65 Father, healthy, age 70 No family history of colon cancer or other malignancies ROS: Negative for weight loss Negative for fever, chills Negative for rash Negative for joint complaints PHYSICAL EXAMINATION: Generally well-appearing man VSS: Afebrile HEENT: PERRLA, EOMI, Tympanic membranes clear Throat clear Neck: Supple, no bruits Heart: RRR without murmurs, rubs, or gallops Lungs: CTA Abdomen: Benign, no tenderness, no hepatosplenomegaly Extremities: No cyanosis, clubbing, or edema GU: Normal male genitalia No testicular masses Rectal: Heme-negative, brown stool Neuro: Reflexes equal and symmetrical throughout; no sensory or motor deficits noted LABORATORY: CBC: WBC 8,500 with 50% polys, 50% lymphs Hemoglobin: 13 mg/dL (after transfusion last week) Platelets: 275,000 Electrolytes: Normal Liver function tests: ALT 40 U/L; AST 23 U/L; alkaline phosphatase 180 mU/mL; Albumin 3.6 mg/dL; total bilirubin 0.5 mg/dL Based on the findings, which of the following is the next step in management? A) Continue routine colonoscopy on a yearly basis unless another exacerbation occurs; then repeat colonoscopy at that time. B) Reinstitute maintenance therapy and repeat colonoscopy in 6 months to see if the "dysplasia" is just a side effect of his exacerbation. C) Recommend referral to surgery. D) Reinstitute maintenance therapy, assuming that he will continue to do well long-term based on his relapse rate of every 10 years. E) Repeat colonoscopy in 2 weeks.

Answer C. Recommend referral to surgery. Explanation Even though he has been doing well until recently, the finding of high-grade dysplasia in flat mucosa indicates that colon cancer is possibly imminent, and removal of his complete colon will be curative. It is difficult to think about this in someone who has had relatively mild ulcerative colitis. Additionally, if you had found a mass lesion that showed dysplasia, complete colectomy would be indicated. Once you are at this stage, repeat colonoscopy is not helpful and will just prolong the inevitable. Likewise, reinstituting medical therapy would run the risk of allowing the high-grade dysplasia to progress.

An 80-year-old African-American man with a history of blood transfusions in 1989 after a motor vehicle accident is noted to have an ALT of 300 while on lipid-lowering therapy. He denies any complaints of icteric illness, or any family history of liver disease. His workup is remarkable for a +hepatitis C antibody with a viral load of 1.6 million IU/mL. His genotype is 1a. An ultrasound of the liver shows a completely normal liver echotexture with a few small 3- to 4-mm gallstones. His bilirubin and alkaline phosphatase are normal. The next appropriate step would be which of the following? A) Observation only. B) Stop atorvastatin. Refer for cholecystectomy. C) Stop atorvastatin and follow ALT levels. Observation only for chronic hepatitis C. D) Start peginterferon and ribavirin only. E) Refer for liver biopsy.

Answer C. Stop atorvastatin and follow ALT levels. Observation only for chronic hepatitis C. Explanation 1) Regarding the hepatitis C infection: This gentleman is likely too old to undergo treatment for viral hepatitis. The current guidelines do not have an absolute age cut-off, but at 80 years old, he likely will have many more complications from the therapy than no treatment, based on his current burden of illness from his disease, which is minimal. Also, he likely has slow progression disease with the only abnormality being an elevated AST. If you were to treat, the current guidelines recommend peginterferon combined with ribavirin and a protease inhibitor (either boceprevir or telaprevir). He likely only needs observation of LFTs at this point. The gallstones are asymptomatic and do not require intervention. 2) Regarding the atorvastatin. This patient has a chronic HCV infection that is probably causing his increased ALT. Statins typically have a lowering effect on ALT in patients with hepatitis C! Also, statins have not been shown to cause liver enzymes elevation any more often than placebo. Even so, current guidelines recommend stopping or decreasing the dose of a statin when level jumps > 3 x NL. Levels are then followed to see what the result is.

A 30-year-old Hispanic male with history of Crohn disease presents for usual routine checkup. He has been doing very well on his current regimen, which includes sulfasalazine and metronidazole. Initially during the office visit, he has no complaints. However, during the end of your examination, he jokes that he and his wife have been trying to get pregnant during the last year without any luck. He says that they have been reading suggestions in books and on the Internet without any luck. His wife is healthy—they have had 2 children, now ages 8 and 7. His Crohn's was diagnosed 6 years ago, and he essentially has been in remission for 5 years on his current therapy. He asks if his Crohn's could be related to the inability to conceive. PAST MEDICAL HISTORY: As above; before diagnosis of Crohn's, had intermittent diarrhea for 5-6 years MEDICATIONS: Sulfasalazine 1 gram PO bid Metronidazole 500 mg PO bid SOCIAL HISTORY: Works as photographer Lives with wife and 2 children in El Paso, TX FAMILY HISTORY: Non-contributory REVIEW OF SYSTEMS: Sexual libido has been normal No problems with erections He wears "briefs" No change in ejaculate No rash No urinary complaints PHYSICAL EXAMINATION: Muscular athletic male in no distress BP 125/62, P 56, RR 12, Temp 98.3° F HEENT: PERRLA, EOMI, Tympanic membranes clear Throat clear Neck: Supple, no masses Heart: RRR without murmurs, rubs, or gallops Lungs: CTA Abdomen: Bowel sounds present in all 4 quadrants, no tenderness, no hepatosplenomegaly Extremities: No cyanosis, clubbing, or edema GU: Normal male genitalia, Tanner V staging No testicular masses Rectal: Heme-negative, tone normal Neurological: Reflexes equal and symmetrical throughout; no sensory or motor deficits noted LABORATORY: CBC: Normal Electrolytes: Normal Liver function tests: Normal Semen analysis: Volume 4 mL; pH 7.5; sperm count 10 million/mL with 85% motile (Normal values volume 2-5 mL; pH 7.2-8.0; sperm count 70-150 million/mL; sperm activity > 80% is normal) Based on your history, physical examination, and laboratory studies, which of the following would be the next best course of action? A) Formal urological evaluation. B) Have him wear boxers instead of briefs. C) Stop sulfasalazine and use another agent for control of his disease. D) Fertility testing of his wife. E) Check serum testosterone.

Answer C. Stop sulfasalazine and use another agent for control of his disease. Explanation Sulfasalazine is split into sulfapyridine and mesalamine. The problem is the sulfapyridine can cause reversible infertility in men. The best answer is to stop the sulfasalazine and prescribe another agent or attempt a trial off of therapy, since he has not had a problem in over 5 years. Once he has achieved "success," then you could restart the sulfasalazine. Formal urologic evaluation is not indicated at this point; we have found that his sperm count is abnormally low, which explains why they are not conceiving. His wife does not need to undergo any testing. Also, since we have a reversible cause for his infertility and he is "functioning" properly, it is unnecessary to proceed with further workup including testosterone levels. It wouldn't hurt to try boxers instead of briefs, but it is unlikely to make a difference in this case while his sperm count returns to normal.

A 28-year-old man presents to your office with a complaint of abdominal pain. He reports a history of generalized crampy pain occurring several times per week that has gone on for several years. He reports that he occasionally has diarrhea that accompanies the pain. Further questioning elicits that he has had mild fatigue that has rarely interfered with his daily activities. He reports that he participated in a fast with his co-workers in attempt to "cleanse" himself by abstaining from solid foods for 2 weeks. He stated that his symptoms resolved completely but subsequently returned after resuming his regular diet. His physical examination is otherwise unremarkable. You obtain the following laboratory studies: WBC: 9.2/uL (4.0-10 x 109/L) Hgb: 13.8 g/dL (14-17g/dL) Hct: 39.5% (41-51%) Plt: 298/uL (150-350 x 109/L) MCV: 75 fL (80-100 fL) Ferritin: 12 ng/mL (15-200 ng/mL) TIBC: 385 ug/dL (250-460 ug/dL) Serum Iron: 44 ug/dL (60-160 ug/dL) Fecal occult blood testing: Negative Anti-tissue Transglutaminase Antibody: Positive What is the next best step in the management of this patient? A) Iron replacement therapy B) Initiation of gluten-free diet C) Upper EGD with small bowel biopsy D) Endomysial antibody testing E) Colonoscopy

Answer C. Upper EGD with small bowel biopsy Explanation Celiac sprue classically presents with foul-smelling diarrhea with steatorrhea and abdominal pain. However, many adult patients have subclinical disease and do not present with classic symptoms. One common manifestation of celiac disease is iron deficiency that is typically mild. The presence of iron deficiency in addition to his symptoms increases the likelihood of celiac sprue and should prompt testing for the presence of anti-tissue transglutaminase antibodies with a sensitivity of testing around 94-98%. Once confirmed, small bowel biopsy should be performed to confirm the diagnosis. Endomysial antibody testing is specific for celiac disease, but biopsy is required for confirmation. Iron replacement will not be likely to improve his deficiency as the underlying cause will be left untreated. A gluten-free diet should not be initiated empirically until biopsy confirmation, as this can confound both the laboratory and biopsy studies. A colonoscopy would be indicated to pursue a diagnosis of inflammatory bowel disease. The absence of fecal occult blood and the positive TTG makes this diagnosis less likely.

A 55-year-old male is referred for evaluation of microcytic anemia. He was recently in the hospital after presenting to the emergency department with a Hgb of 7 and an MCV of 65. He had noticed gradual weakness for 3 months prior to this but thought it was due to a viral syndrome. He denied any specific complaints and, in fact, denied any overt rectal bleeding or change in bowel movements. While he was in the hospital, he had an upper endoscopy, which was normal. Colonoscopy was also performed with good visualization of the colon to the cecum, again without any abnormalities. He received 2 units of packed RBCs and was discharged on oral iron. PAST MEDICAL HISTORY: Otherwise unremarkable; he is on no medications; he does not take aspirin or NSAIDs FAMILY HISTORY: Negative for any specific diseases of the GI tract SOCIAL HISTORY: Negative REVIEW OF SYSTEMS: Significant for fatigue and dyspnea on exertion; otherwise negative PHYSICAL EXAMINATION: Healthy 55-year-old; although he was pale at the time of admission, his color has now returned after the blood transfusion At this point, you recommend which of the following? A) Trial of estrogen empirically for presumed AV malformation bleeding B) CT scan of the abdomen and pelvis C) Upper GI small bowel series D) Ultrasound of the liver E) Tagged RBC bleeding scan

Answer C. Upper GI small bowel series Explanation This is a 55-year-old with occult GI bleeding that has been severe enough to cause profound anemia requiring blood transfusions. No source was found for the bleeding in either the upper or the lower tract. Likely, this is due to a small bowel bleeding source. Although it is possible that this could be due to an AV malformation, which is the most common source of small bowel bleeding, he is younger than most patients with this type of bleeding. Therefore, one thinks of other possibilities, which includes tumors of the small bowel, which can present with bleeding. For this indication, one could do either an enteroscopy or an upper GI small bowel series. The x-ray has the advantage of visualizing the entire bowel, although it's not as accurate for the upper region of the jejunum as enteroscopy would be. It is controversial whether estrogen would have helped AV malformation bleeding anyway, and this medicine is very poorly tolerated in men. There is no indication that bleeding rate is rapid enough for a tagged RBC scan to be beneficial. If the GI series demonstrates a tumor in the small bowel, then a CT scan would be appropriate, but not at this point.

A 23-year-old man comes to your office with complaints of intermittent dysphagia. He says it occurs after swallowing a large amount of food—especially if he takes a big bite of steak and doesn't chew it very well. It has never occurred with liquids. The dysphagia occurs every 1-2 months and then goes away. He has had some heartburn in the past but none recently. He denies chest pain, nausea, vomiting, or sore throat. Social History: He works as an auto mechanic and is a non-smoker. Family History: Family history is significant for no known malignancies. Which of the following tests listed would be the most appropriate diagnostic test? A) H. pylori testing. B) No procedure is indicated. C) Upper endoscopy. D) Manometry. E) 24-hour pH probe.

Answer C. Upper endoscopy. Explanation He has dysphagia for solids and not liquids—this suggests anatomic obstruction. In younger patients, it is usually due to a Schatzki ring (lower esophageal ring), while in older patients it is usually due to cancer. Schatzki rings are common (1/7 people have them) and are usually asymptomatic unless the lumen becomes smaller than 13 mm in diameter. The non-progressive nature is an important clue in this history. If he had a peptic stricture, it would slowly progress from solids to eventually include liquids.

A 78-year-old nursing home patient presents with a readmission to the hospital for diarrhea. She was hospitalized one month ago and was found to have stool that was positive for C. difficile toxin. She was treated with metronidazole but experienced severe nausea and vomiting. She was sent home on this regimen, but, according to the nursing home staff, she was unable to complete her course of therapy because of the vomiting. The frequency of the stools improved but never returned to baseline. The diarrhea became worse the day prior to admission. Past Medical History: Significant for several cerebrovascular accidents in the past. She has recurrent urinary tract infections that have required numerous courses of antibiotics over the past 2 years. She has limited activity. She is able to feed herself without assistance. She enjoys watching "Wheel of Fortune" each evening. Physical Exam: She is in no apparent distress. The exam is unremarkable for this elderly woman. Abdomen is slightly distended but without tympany. The bowel sounds seem hyperactive. Initial studies show stool that is positive for fecal leucocytes as well as positive for the toxin for C. difficile. Her WBC count is 16,500. Based on these findings, which of the following regimens would be most appropriate next course of action? A) Amoxicillin with intravenous gentamicin B) A regimen of probiotics C) Vancomycin 250 mg PO qid x 14 days D) Intravenous metronidazole in conjunction with promethazine (Phenergan®) to prevent the vomiting E) Intravenous fluid with oral mesalamine

Answer C. Vancomycin 250 mg PO qid x 14 days Explanation For C. difficile-related colitis, the preferred treatment is metronidazole. However, there are patients who cannot tolerate metronidazole, and vancomycin has a similar efficacy. The main problem with C. difficile is the recurrence, even with appropriate antibiotic therapy, in up to 30% of patients. The role of probiotics is an interesting one but not well established at this time. There certainly may be a role for probiotics in patients in between treatments if they are suffering with recurrent infections. Intravenous metronidazole could be appropriately used if the pseudomembranous colitis were so severe that the patient had an ileus and could not tolerate PO treatment—but this would probably be the only appropriate setting for intravenous metronidazole.

A 60-year-old Caucasian male has a long history of reflux symptoms. Most of these symptoms are well controlled with omeprazole every morning. He also has adult-onset asthma, and this is, in fact, the biggest problem for this patient at this time. The asthma started at about the age of 40. It has progressively worsened and is difficult to control with inhalant therapy. He has required frequent courses of prednisone, but he is becoming less responsive to this therapy as well. Social History: The patient has a good diet without any bad habits. Review of Systems: Positive for nocturnal regurgitation and occasional waking with choking sensation Upper GI series shows a small hiatal hernia and reflux. EGD: Demonstrated linear esophageal erosion but no suspicion of Barrett esophagus Which of the following treatments do you recommend? A) Increase the omeprazole to twice a day. B) Evaluate for anti-reflux surgery. C) Intensify the asthma regimen with chronic steroids. D) Empiric fluconazole for possible candida esophagitis. E) Add an H2 blocker at bedtime to the morning proton pump inhibitor.

B. Evaluate for anti-reflux surgery. Explanation Prednisone may lead to Candida in the otherwise immunocompetent host, but there is no evidence of this at endoscopy; therefore, no reason to treat for this. The use of omeprazole may indeed be effective in this patient; however, given his relatively young age and the esophageal damage despite once-a-day proton pump inhibitor usage, this patient would be a good candidate for anti-reflux surgery. There is a high likelihood that the reflux is involved in this patient's asthma, and the surgery is going to have a better result than even twice-a-day proton pump inhibitors. Adding an H2 blocker at night to the morning proton pump inhibitor may be beneficial for an occasional breakthrough, but it has little value for long-term management. Data has shown that symptomatic reflux can exacerbate asthma and treatment is beneficial, especially if the asthma occurs at night. Newer data shows that people with asthma commonly have asymptomatic reflux and that treatment of asymptomatic reflux does NOT improve asthma symptoms or control.

A 64-year-old man presents to the emergency department complaining of central abdominal pain that has progressively worsened over the last 2 days. He reports associated nausea and vomiting that worsens with PO intake. His past medical history includes hypertension and hyperlipidemia. Past surgical history includes a cholecystectomy 10 years ago. He smokes 1/2 ppd and drinks 1-2 beers/day. Medications include lisinopril, hydrochlorothiazide, and simvastatin. On physical exam, temperature is 37.5° C, heart rate 105/min, blood pressure 135/83, respirations 22/min. He appears in mild distress secondary to pain. Abdominal exam reveals a distended abdomen with tenderness to palpation greatest in the epigastric region. Abdominal skin is discolored as shown in the image. Heart and lung exams are otherwise unremarkable. Laboratory studies are significant for a lipase of 8,053 units/L. Which of the following is the most likely precipitant of the patient's current symptoms? A) Tobacco use B) Hypertriglyceridemia C) Cholelithiasis D) Alcohol E) Hydrochlorothiazide

Answer D. Alcohol Explanation The patient in this question has acute pancreatitis, manifested by abdominal pain with associated nausea/vomiting and elevated lipase level. Gallstones are the most common precipitant of acute pancreatitis, but this patient has a history of remote cholecystectomy that makes this answer unlikely. After gallstones, alcohol is the next most common cause, accounting for up to 30% of cases in the United States. Tobacco is more commonly associated with chronic pancreatitis rather than acute pancreatitis. Hydrochlorothiazide has been linked with elevations in serum calcium and pancreatitis, but the incidence is much more uncommon compared to alcohol and gallstone pancreatitis. Hypertriglyceridemia is an infrequent cause of acute pancreatitis, accounting for approximately 1-3% of episodes.

A 52-year-old man presents for routine colonoscopy. PAST MEDICAL HISTORY: Insulin-dependent diabetes for 4 years Hypertension for 3 years Hyperlipidemia for 2 years SOCIAL HISTORY: Works as a waiter Smokes 2 ppd of cigarettes Drinks 3-4 beers nightly FAMILY HISTORY: Father 80, a minister with diabetes Mother 75, lives in Florida, osteoporosis Sister 55 with SLE REVIEW OF SYSTEMS: No headache No sore throat No fevers No night sweats No chest discomfort Chronic "smoker" type cough; worse in the morning on awakening No weight loss PHYSICAL EXAMINATION: BP 130/60, P 90, RR 18, Temp 99.0° F HEENT: PERRLA, EOMI, sclera non-icteric Neck: Supple; no bruits; no masses Heart: RRR with new II/VI systolic flow murmur (old) Lungs: Clear to auscultation Abdomen: Mild epigastric tenderness to deep palpation; positive bowel sounds in all 4 quadrants, no hepatosplenomegaly Extremities: No cyanosis, clubbing, or edema GU: Normal male; no masses Rectal: Deferred because about to have colonoscopy PROCEDURES: Colonoscopy: 1 suspicious polyp was found in the right colon near the sigmoid junction; completely resected. He returns in 1 week for his pathology report. These findings show: The polyp was completely excised and submitted in toto for pathological examination. The polyp was fixed and sectioned so that it was possible to accurately determine the depth of invasion, grade of differentiation, and completeness of the excision of the polyp. Unfortunately, carcinoma is found in the tissue. The cancer is well differentiated. There is no vascular or lymphatic involvement. The margin of the excision is not involved. Repeat colonoscopy in 3 months shows no residual abnormal tissue at the polypectomy site. Based on your findings, which of the following should be your recommendation? A) Repeat colonoscopy in 6 months. B) Determine CEA levels. C) Refer for surgical intervention and chemotherapy. D) Because the incidence of recurrent cancer is small, no other laboratory or imaging studies are indicated for this patient; follow-up should proceed as with benign adenomas. E) Repeat colonoscopy in 3 months.

Answer D. Because the incidence of recurrent cancer is small, no other laboratory or imaging studies are indicated for this patient; follow-up should proceed as with benign adenomas. Explanation He meets all of the favorable prognostic criteria outlined by the American College of Gastroenterology: 1. The polyp was completely excised and submitted in toto for evaluation. 2. The pathology lab fixed and sectioned the material to accurately determine the depth of invasion, grade of differentiation, and completeness of excision. 3. The cancer is not poorly differentiated. 4. There is no vascular or lymphatic involvement. 5. The margin of excision is not involved. These patients with favorable prognostic criteria should have follow-up colonoscopy in 3 months to check for residual abnormal tissue at the polypectomy site if the polyp is sessile. After 1 negative examination, care can revert to standard surveillance as performed for patients with benign adenomas. Because the incidence of recurrent cancer is small, no other follow-up laboratory or imaging studies are indicated.

A 35-year-old Caucasian female has had Crohn disease since age 19. She has had one prior surgery, which was the resection of the cecum and terminal ileum at age 23. She has required a number of tapering courses of prednisone in early years but only twice in the past 5 years. She describes 2-3 soft stools per day. She has occasional abdominal cramps. She considers herself fairly well controlled right now with regard to the Crohn disease. Her current medication is mesalamine 2.4 g/day. Review of Systems: She denies any joint or bone pain. She does not have fever. Her weight is stable. Physical Examination: There is slight fullness in the right lower quadrant without a definite mass. Colonoscopy last year demonstrated recurrent disease at the anastomosis, extending about 15 cm into the ileum. Which of the following would be the most appropriate next step at this time? A) Initiate azathioprine and, 3 months later, check bone mineral density. B) Bone mineral density at age 40, but for now maximize calcium intake. C) Empiric weekly alendronate. D) Bone mineral density now.

Answer D. Bone mineral density now. Explanation Due to both her past steroid use and active disease possibly interfering with absorption, this patient is at risk for osteoporosis. She is somewhat young at age 35, but density should be checked out now. There is no reason to initiate azathioprine for this woman because clinically she is doing well on the mesalamine.

A 60-year-old Caucasian male presents to your office with a chief complaint of dysphagia. Dysphagia has been present for about 2 years. He notes difficulty swallowing both solids and liquids, which has been slightly progressive over the 2-year period. He also notes some nocturnal regurgitation. His primary care doctor saw him a year ago and gave him omeprazole. However, this has not improved the symptoms. Past Medical History: Remarkable for coronary artery disease and cardiomyopathy, with an ejection fraction of 20%. He's had a CVA in the past although no residual impairment. He also has a long history of COPD due to both past and current cigarette use. Upper GI is ordered, which demonstrates a dilated esophagus that is tapered at the junction to the stomach. EGD shows the same findings. There is a dilated esophagus with retained food and smooth tapering at the distal esophagus. There is no evidence of any mass effect at the distal esophagus. The stomach and duodenum are normal. Which of the following is the best treatment recommendation for this individual? A) Pneumatic dilatation B) Laparoscopic myotomy C) Bougie dilatation D) Botox injection into the lower esophageal sphincter E) Esophageal resection

Answer D. Botox injection into the lower esophageal sphincter Explanation Treatment of achalasia depends on the patient's characteristics as well as the physician preferences. For the patient who is a good operative risk, the two most often recommended treatments are pneumatic dilatation and laparoscopic myotomy. Laparoscopic myotomy, being a surgical procedure, is more invasive, but some physicians feel it has a better long-term outcome in terms of patient satisfaction. Pneumatic dilatation is associated with an acceptable complication rate; however, there is a 5% perforation risk. For patients with high surgical risk, a trial with botulinum toxin is done. With COPD and severe cardiomyopathy (EF < 20%), this patient is a very poor surgical candidate so botulinum toxin injection is the most appropriate treatment. This would gives the patient between 6 and 12 months of relief, at which time the patient could have another Botox injection. Botox injection is safe and can be done as an outpatient.

A 68-year-old Caucasian female presents to her primary care physician with a chief complaint of weakness and fatigue. This has been present for 2 months, although she initially attributed this to the stress of the holidays. She has always been healthy and active. She denies any abdominal pain, change in bowel habits, chest pain, or pulmonary symptoms. However, she does admit to some dyspnea on exertion. Past Medical History: Otherwise healthy. Her only medication is estrogen replacement. She has had a remote hysterectomy. Family History: Unremarkable Social History: Unremarkable Review of Systems: She denies any melena or rectal bleeding. She has occasional nocturnal reflux after a late meal, but this happens only once a month and has been completely relieved by OTC famotidine. Physical Examination: She is somewhat pale, but in no apparent distress. HEENT: Unremarkable. There is no lymphadenopathy. Chest: Clear Cardiac Exam: Normal Abdomen: Soft and nontender. There are no palpable masses. There is no enlargement of liver or spleen. Rectal Exam: Stool is brown and heme-positive. Laboratory: Hgb 9.6, MCV 72, LFTs are normal. Which of the following should you order next? A) EGD B) Tagged RBC bleeding scan C) Upper GI small bowel series D) Colonoscopy

Answer D. Colonoscopy Explanation This 68-year-old woman has a new microcytic anemia and heme-positive stool, reflecting occult gastrointestinal blood loss. In general, when a person in this age group presents with occult GI bleeding, the foremost concern is that of ruling out a colonic malignancy. Therefore, a colonoscopy is the first test to order. If she was having frequent reflux or had a long history of symptomatic reflux, then one might even consider doing an upper endoscopy at the same time. However, her reflux symptoms are very rare and probably fall within normal symptomatology. There is no indication that she is bleeding at a rate rapid enough for a tagged nuclear medicine scan to be helpful, since this study does require a fairly brisk and ongoing bleeding. There is no value to the upper GI small bowel series, unless she is completely negative at the colonoscopy and has a subsequent EGD that is normal as well. As an initial study, this would not be appropriate.

A 30-year-old male presents for a routine examination. He has no specific complaints. He has just moved to the area with his job. Past Medical History: Negative, and his only medications are multivitamins. He exercises daily and doesn't smoke cigarettes or drink alcohol. Family History: He notes that his father had colon cancer at age 55, and in fact, he is now undergoing chemotherapy. He had a paternal uncle who died from colon cancer at age 42. His brother, who is 35 years old, recently had a colonoscopy, at which time several polyps were removed. The patient doesn't know the histology of those polyps, just that they were small. Which of the following studies would you recommend? A) Air-contrast barium enema now B) Flexible sigmoidoscopy now combined with fecal occult blood testing every year and a colonoscopy at age 40 C) Colonoscopy at age 40 D) Colonoscopy now

Answer D. Colonoscopy now Explanation The history that is described is suggestive of hereditary non-polyposis colon cancer (HNPCC), or Lynch syndrome. There is an uncle who died from colon cancer at a remarkably young age. The father, with cancer at age 55, is slightly younger than normal, and this might not raise any suspicion by itself. Certainly the situation of the brother, who is only a few years older and has polyps, is worrisome. For cases of HNPCC, it is very reasonable to begin colonoscopy at age 25. These patients often have right-sided colon cancer; therefore, a flexible sigmoidoscopy is of no value. Fecal occult blood testing is not reliable and is often negative, even in the face of known cancer.

A 70-year-old female complains of vague abdominal discomfort. She has never had any severe pain. This discomfort has been present for several months. It is located in the middle-to-upper abdomen. PAST MEDICAL HISTORY: Unremarkable; she is healthy and does not take medications except for a multi-vitamin SOCIAL HISTORY: She is a very healthy woman who enters 10k races on a regular basis FAMILY HISTORY: Positive for brother with colon cancer ROS: She denies any jaundice, weight loss, or change in bowel habits; she has not had any back pain or itching LABS: CBC and LFTs are all within normal limits You order an abdominal CT scan, which demonstrates a 7-cm cyst in the tail of the pancreas. Which of the following should you recommend? A) Start pancreatic enzymes, because this is likely a pseudocyst from occult chronic pancreatitis. If the cyst is persistent 3 months later, then recommend percutaneous drainage by the radiologist. B) Consult surgery for elective resection of the tail of the pancreas with removal of the cyst. C) CT-directed needle aspiration. If the fluid is negative for cytology, then you can reassure the patient. D) Check CA19-9 level; if normal, reassure the patient and recommend no further workup.

This patient does indeed have a cyst in the pancreas, but it is unlikely that this is a pseudocyst, given that she has never had pain or symptoms to suggest either acute or chronic pancreatitis. In this case, one must consider the possibility of a cystic neoplasm, which can be either a cystadenoma or a cystadenocarcinoma or, rarely, cystic islet cell tumors. A CA19-9 level might be elevated in cases of pancreatic adenocarcinoma, but it would not be helpful in the cystic neoplasms. Likewise, CT-directed needle aspiration often yields negative cytology even in the case of cystadenocarcinoma. Since this woman is healthy, she should undergo resection of the tail of the pancreas, which is a relatively easy procedure and will be curative in this case.


Set pelajaran terkait

FBLA Banking and Financial Systems

View Set

084 Social Studies (Secondary) - General 2.0

View Set

chpt 13 psychology test study guide

View Set

Chapter 13: Abstract Classes and Interfaces

View Set

Art from China's Bronze Age and Cultural Innovations

View Set

Cyber 3100 Principles of Info Security Chapter 1

View Set

Nutrition exam 3 and final quizzes

View Set

Biomedical Technology 1 Final Review

View Set